Sie sind auf Seite 1von 73

INSURANCE CASES and a family friend by the name of Jose Manuel sustained

physical injuries of varying degrees. Amor Songco, 42-year-old


1. Fieldmen's Insurance Co., Inc. v. Vda. de Songco son of deceased Federico Songco, testifying as witness,
G.R. No. L-24833 [September 23, 1968], 134 PHIL 90- declared that when insurance agent Benjamin Sambat was
96 inducing his father to insure his vehicle, he butted in saying:
'That cannot be, Mr. Sambat, because our vehicle is an 'owner'
private vehicle and not for passengers,' to which agent Sambat
DOCTRINE: "The contract of insurance is one of perfect good replied: 'whether our vehicle was an 'owner' type or for
faith (uberrima fides) not for the insured alone, but equally so passengers it could be insured because their company is not
for the insurer; in fact, it is more so for the latter, since its owned by the Government and the Government has nothing to
dominant bargaining, position carries with it stricter do with their company. So they could do what they please
responsibility." whenever they believe a vehicle is insurable'. The defendant
company did not even care to rebut Amor Songco's testimony
FACTS: Federico Songco of Floridablanca, Pampanga, a man by calling on the witness-stand agent Benjamin Sambat, its
of scant education, being only a first grader, owned a private Pampanga Field Representative.
jeepney with Plate No. 41-289 for the year 1960. On September
15, 1960, as such private vehicle owner, he was induced ISSUE: Can FIELDMEN'S Insurance Co., Inc., be held liable
by FIELDMEN'S Insurance Company Pampanga agent under a common carrier insurance policy even though what was
Benjamin Sambat to apply for a Common Carrier's insured was a private vehicle and not a common carrier?
Liability Insurance Policy covering his motor vehicle. Upon
paying an annual premium of P16.50, RULING: YES. It is now beyond question that where
defendant FIELDMEN'S Insurance Company Inc. issued on inequitable conduct is shown by an insurance firm, it is
September 19, 1960, Common Carriers Accident "estopped from enforcing forfeitures in its favor, in order to
Insurance Policy, the duration of which will be for one (1) year, forestall fraud or imposition on the insured." After
effective September 15, 1960 to September 15, 1961. On petitioner FIELDMEN'S Insurance Co., Inc., had led the insured
September 22, 1961, the defendant company, upon payment of Federico Songco to believe that he could qualify under the
the corresponding premium, renewed the policy by extending common carrier liability insurance policy, and to enter into
the coverage from October 15, 1961 to October 15, 1962. This contract of insurance paying the premiums due, it could not,
time Federico Songco's private jeepney carried Plate No. J- thereafter, in any litigation arising out of such representation, be
68136- Pampanga – 1961. During the effectivity of the renewed permitted to change its stand to the detriment of the heirs of the
policy, the insured vehicle while being driven by insured. As estoppel is primarily based on the doctrine of good
Rodolfo Songco, a duly licensed driver and son of Federico (the faith and the avoidance of harm that will befall the innocent party
vehicle owner) collided with a car in the municipality of Calumpit, due to its injurious reliance, the failure to apply it in this case
province of Bulacan, as a result of which mishap would result in a gross travesty of justice.
Federico Songco and Rodolfo Songco died, That is all that needs be said insofar as the first alleged error of
Carlos Songco (another son), the latter's wife, Angelita Songco, respondent Court of Appeals is concerned, petitioner being
adamant in its far-from-reasonable plea that estoppel could not may not change one whit, his participation in the 'agreement'
be invoked by the heirs of the insured as a bar to the alleged being reduced to the alternative to 'take it or leave it' labelled
breach of warranty and condition in the policy. It would now rely since Raymond Saleilles 'contracts by adherence' (contracts d'
on the fact that the insured owned a private vehicle, not a adhesion), in contrast to these entered into by parties
common carrier, something which it knew all along, when not bargaining on an equal footing, such contracts (of which policies
once but twice its agent, no doubt without any objection in its of insurance and international bills of lading are prime example)
part, exerted the utmost pressure on the insured, a man of scant obviously call for greater strictness and vigilance on the part of
education, to enter into such a contract. courts of justice with a view to protecting the weaker party from
abuses and imposition, and prevent their becoming traps for the
Nor is there any merit to the second alleged error of respondent unwary (New Civil Code, Article 24; Sent. of Supreme Court of
Court that no legal liability was incurred under the policy by Spain, 13 Dec. 1934, 27 February 1942)."
petitioner. Why liability under the terms of the policy was
inescapable was set forth in the decision of respondent Court of The conclusion that inescapably emerges from the above is the
Appeals. Thus: "Since some of the conditions contained in the correctness of the decision of respondent Court of Appeals
policy issued by the defendant-appellant were impossible to sought to be reviewed. For, to borrow once again from the
comply with under the existing conditions at the time and language of the Qua Chee Gan opinion: "The contract
'inconsistent with the known facts,' the insurer 'is estopped from of insurance is one of perfect good faith (uberrima fides)
asserting breach of such conditions.' From this jurisprudence, not for the insured alone, but equally so for the insurer; in
we find no valid reason to deviate and consequently hold that fact, it is more so for the latter, since its dominant
the decision appealed from should be affirmed. The injured bargaining, position carries with it stricter
parties, to wit, Carlos Songco, Angelito Songco and Jose responsibility." This is merely to stress that while the morality
Manuel, for whose hospital and medical expenses the of the business world is not the morality of institutions of
defendant company was being made liable, were passengers rectitude like the pulpit and the academe, it cannot descend so
of the jeepney at the time of the occurrence, and low as to be another name for guile or deception. Moreover,
Rodolfo Songco, for whose burial expenses the defendant should it happen thus, no court of justice should allow itself to
company was also being made liable was the driver of the lend its approval and support.
vehicle in question. Except for the fact, that they were not fare-
paying passengers, their status as beneficiaries under the 2. National Development Company v. Madrigal Wan Hai
policy is recognized therein." Lines Corp.
G.R. No. 148332, [September 30, 2003], 458 PHIL 1038-1056
"This rigid application of the rule on ambiguities has become
necessary in view of current business practices. The courts DOCTRINES:
cannot ignore that nowadays monopolies, cartels and  A contract of adhesion is one in which one of the parties
concentrations of capital, endowed with overwhelming imposes a ready-made form of contract, which the other party
economic power, manage to impose upon parties dealing with may accept or reject, but which the latter cannot modify. In other
them cunningly prepared 'agreements' that the weaker party words, in such contract, the terms therein are fixed by one party,
and the other party has merely "to take it, or leave it." Thus, it refusal to reimburse respondent of the amount it paid to the US
can be struck down as void and unenforceable for being IRS, the latter filed a complaint against the former for
subversive of public policy, especially when the will of the reimbursement and damages. The trial court ruled in favor of
dominant party is imposed upon the weaker party and the latter respondent. On appeal, the Court of Appeals affirmed the trial
is denied the opportunity to bargain on equal footing. court's decision with modification. Hence, this petition.|||
 Contracts of adhesion—wherein one party imposes a ready-
made form of contract on the other—are not entirely prohibited.
ISSUE: Whether the Negotiated Sale Guidelines and the
The other party is free to reject it entirely; if he adheres, he gives
Proposal Letter Form constitute a contract of adhesion.
his consent. Nevertheless, the inequality of bargaining positions
and the resulting impairment of the other party's freedom to RULING: YES. The Supreme we agree with both lower courts
contract necessarily call upon us to exercise our mandate as a that the Negotiated Sale Guidelines and the Proposal Letter
court of justice and equity. Indeed, we have ruled that contracts Form constitute a contract of adhesion.
of such nature "obviously call for greater strictness and vigilance
on the part of the courts of justice with a view to protecting the A contract of adhesion is one in which one of the parties
weaker party from abuses and imposition and prevent their imposes a ready-made form of contract, which the other party
becoming traps for the unwary. may accept or reject, but which the latter cannot modify. In other
words, in such contract, the terms therein are fixed by one party,
FACTS: The National Development Company, petitioner, is a and the other party has merely "to take it, or leave it." Thus, it
government-owned and controlled The National Shipping can be struck down as void and unenforceable for being
Corporation of the Philippines (NSCP) is a wholly- subversive of public policy, especially when the will of the
owned subsidiary of petitioner offering shipping services for dominant party is imposed upon the weaker party and the latter
containerized cargo between the Far East ports and the U.S. is denied the opportunity to bargain on equal footing.
West Coast. Petitioner's Board of Directors approved the
privatization plan of the NSCP. In May 1993, the Board offered It must be stressed, however, that contracts of adhesion are not
for sale to the public its one hundred percent (100%) stock strictly against the law. In Ong Ku vs. Court of Appeals and Pan
ownership in NSCP worth P150,000.00, as well as its three (3) American World Airways, Inc. vs. Intermediate Appellate Court,
ocean-going vessels (M/VNational Honor, M/V National it was held that contracts of adhesion — wherein one party
Pride and M/V National Dignity). Private respondent acquired in imposes a ready-made form of contract on the other — are not
a public bidding petitioner's stock ownership in NSCP, as well entirely prohibited. The other party is free to reject it entirely; if
as its three vessels. Subsequently, respondent received from he adheres, he gives his consent.
the US Department of Treasury, Internal Revenue Service (US Nevertheless, the inequality of bargaining positions and the
IRS), a notice of final assessment against NSCP for deficiency resulting impairment of the other party's freedom to contract
taxes on gross transportation income derived from US sources. necessarily call upon us to exercise our mandate as a court of
Anxious that the delay in the payment of the deficiency taxes justice and equity. Indeed, we have ruled that contracts of such
may hamper its shipping operations overseas, respondent nature "obviously call for greater strictness and vigilance on the
assumed and paid petitioner's tax liabilities. For petitioner's part of the courts of justice with a view to protecting the weaker
party from abuses and imposition and prevent their becoming not invalid per se and that it has, on numerous occasions,
traps for the unwary." upheld the binding effect thereof.
 While contracts of adhesion may be struck down as void and
In the case at bar, the Negotiated Sale Guidelines and Proposal unenforceable for being subversive of public policy, the same
Letter Form fit the characteristics of a contract of adhesion. On can only be done when, under the circumstances, the weaker
their very face, these documents show that petitioner NDC had party is imposed upon in dealing with the dominant bargaining
control over the terms and conditions of the sale. party and is reduced to the alternative of taking it or leaving it,
|||Clearly, respondent had hardly any say in the terms and completely depriving the former of the opportunity to bargain on
conditions expressed in the Negotiated Sale Guidelines. Other equal footing.
than the price of the offer, respondent was left with little or no  Although in this jurisdiction, contracts of adhesion have been
alternative at all but to comply with its terms. The said contracts consistently upheld as valid per se; as binding as an ordinary
are ready-made form of contracts, the preparation of which was contract, the Court recognizes instances when reliance on such
left entirely to the NSCP. Their nature is that of a contract of contracts cannot be favored especially where the facts and
adhesion. In the case at bar, the acceptance of the Negotiated circumstances warrant that subject stipulations be disregarded.
Sale Guidelines and submission thereof together with the
Proposal Letter Form by a prospective buyer is a required FACTS: KCSI and WG&A Jebsens Shipmanagement,
formality of the bidding. Under this circumstance, the plaintiff, in Inc. (WG&A) entered into, and executed, a Shiprepair
taking such contracts, may not be deemed to have been given Agreement wherein KCSI agreed to carry out renovation and
the opportunity to bargain on equal footing. reconstruction of M/V Superferry 3 (Superferry 3), owned by
Being a contract of adhesion, we reiterate that it is our duty to WG&A, using its (KCSI's) dry docking facilities. Prior to the
apply a strict construction of its terms upon the party who made execution of the Shiprepair Agreement, Superferry 3 was
the same and to construe any ambiguity in such contract already insured by WG&A with Pioneer for US$8,472,581.78.
against its author. It is public policy to protect a party (herein While undergoing repair, Superferry 3 was gutted by fire. WG&A
respondent) against oppressive and onerous conditions. declared the vessel's damage as a "total constructive loss" and
filed an insurance claim with Pioneer. Pioneer paid the
||| insurance claim of WG&A in the amount of US$8,472,581.78.
3. Keppel Cebu Shipyard, Inc. v. Pioneer Insurance and In exchange, WG&A executed a Loss and Subrogation Receipt
Surety Corp. in favor of Pioneer.
G.R. Nos. 180880-81 & 180896-97, [September 18, 2012], Believing that KCSI was solely responsible for the loss of
695 PHIL 169-226 Superferry 3, Pioneer tried to collect the amount of
US$8,472,581.78 form KCSI but it was frustrated.
DOCTRINE: Thus, Pioneer sought arbitration with the Construction Industry
 While greater vigilance is required in determining the validity of Arbitration Commission (CIAC) pursuant to the arbitration
clauses arising from contracts of adhesion, the Court has clause in the Shiprepair Agreement. During the arbitration
nevertheless consistently ruled that contracts of adhesion are
proceedings, an amicable settlement was forged between KCSI a strong and cogent reason to disturb it. It is a hornbook doctrine
and WG&A. Pioneer, thus, stayed on as the remaining claimant. that, save for certain exceptions, the findings of fact of
administrative agencies and quasi-judicial bodies like the CIAC,
The CIAC rendered its Decision finding that both WG&A and
which have acquired expertise because their jurisdiction is
KCSI were equally guilty of negligence which resulted in the
confined to specific matters, are generally accorded not only
fire and loss of Superferry 3. The CIAC also ruled that the
respect, but finality when affirmed by the CA.
liability of KSCI was limited to the amount of P50,000,000.00
pursuant to Clause 20 of the Shiprepair Agreement. Both Be that as it may, the Court, after making its own assiduous
parties appealed to the Court of Appeals (CA). In its final assessment of the case, concurs with the conclusions arrived
disposition of the cases, the CA, through its Amended Decision, at by the tribunals below that the loss of Superferry 3 cannot be
affirmed the decision of the CIAC but deleted its order that attributed to one party alone.
KCSI pay legal interest on the amount due to Pioneer.
WG&A was negligent because, although it utilized the welders
Again, both parties appealed to the Supreme Court. In its of KCSI, it used them outside the agreed area, the restaurant of
Decision, dated September 25, 2009, the Third Division of the the promenade deck. If they did not venture out of the
Court partially granted the appeals of both parties. In granting restaurant, the sparks or the hot molten slags produced by the
the petition of Pioneer, the Court found that KCSI welding of the steel plates would not have reached the
was solely liable for the loss of the vessel and that WG&A combustible lifejackets stored at the deck below.
properly declared the loss of the vessel as constructive total
loss. The Court also declared that Clause 20 of the Shiprepair On the part of KCSI, it failed to secure a hot work permit
Agreement which limited KCSI's liability to the amount of pursuant to another work order. Had this been applied for by the
P50,000,000.00 was invalid. As for the petition of KCSI, the KCSI worker, the hot work area could have been inspected and
Court found merit in KCSI's assertion that the salvage recovery safety measures, including the removal of the combustible
value of the vessel amounting to P30,252,648.09 must be lifejackets, could have been undertaken. In this regard, KCSI is
considered and deducted from the amount KCSI was liable to responsible.
Pioneer. Aggrieved, KCSI moved for the reconsideration. In short, both WG&A and KCSI were equally negligent for the
loss of Superferry 3. The parties being mutually at fault, the
degree of causation may be impossible of rational assessment
ISSUE: To whom may negligence over the fire that broke out as there is no scale to determine how much of the damage is
on board M/V "Superferry 3" be imputed? Is subrogation attributable to WG&A's or KCSI's own fault. Therefore, it is but
proper? If proper, to what extent can subrogation be made? fair that both WG&A and KCSI should equally shoulder the
burden for their negligence.
With respect to the defenses of KCSI that it was a co-assured
RULING: The issue of the conflicting claims between the parties
under Clause 22 (a) of the contract and that its liability is limited
— as to who should be responsible for the loss of Superferry 3
to P50,000,000.00 under Clause 20 of the Shiprepair
— was resolved by the CIAC against both parties. As this finding
Agreement, the Court maintains the earlier ruling on the
of fact by the CIAC was affirmed by the CA, the Court must have
invalidity of Clause 22 (a) of the Shiprepair Agreement.
It cannot, however, maintain the earlier ruling on the invalidity have easily transacted with other ship repairers, which may not
of Clause 20 of the Shiprepair Agreement, which limited KCSI's have included such a provision.
liability to P50,000,000.00. In the September 25, 2009 Decision,
After the signing of the Shiprepair agreement, the record is
the Third Division found Clause 20 of the Shiprepair Agreement
bereft of any other evidence to show that WG&A had protested
invalid, seeing it as an unfair imposition by KCSI, being the
such a provision limiting the liability of KCSI. Indeed, the parties
dominant party, on WG&A. TCASIH
bound themselves to the terms of their contract which became
Basic is the rule that parties to a contract may establish the law between them.
such stipulations, clauses, terms, or conditions as they
While contracts of adhesion may be struck down as void and
may deem convenient, provided they are not contrary to
unenforceable for being subversive of public policy, the same
law, morals, good customs, and public policy. While
can only be done when, under the circumstances, the weaker
greater vigilance is required in determining the validity of
party is imposed upon in dealing with the dominant bargaining
clauses arising from contracts of adhesion, the Court has
party and is reduced to the alternative of taking it or leaving it,
nevertheless consistently ruled that contracts of adhesion
completely depriving the former of the opportunity to bargain on
are not invalid per se and that it has, on numerous
equal footing. This is not the situation in this case.
occasions, upheld the binding effect thereof.
The Court is not unaware of the case of Cebu Shipyard
In its Decision, the Third Division placed great weight in the
Engineering Works, Inc. v. William Lines, Inc., where the Court
testimony of Engr. Elvin F. Bello, WG&A's fleet manager, that
struck down an almost similar provision limiting the liablility of
while he assented to the Shiprepair Agreement, he did not sign
the ship repairer. In the said case, however, the Court found the
the fine-print portion thereof where Clause 20 was found
provision unconscionable not only because the ship repairer
because he did not want WG&A to be bound by them. This
therein was solely negligent in causing the loss of the vessel in
testimony however, was correctly found by the CIAC as clearly
their custody, but also because the limited liability clause sought
self-serving, because such intention of WG&A was belied by its
to be enforced unduly restricted the recovery of the insurer's
actions before, during and after the signing of the Shiprepair
loss of P45,000,000.00 to onlyP1,000,000.00. Careful in not
Agreement.
declaring such a provision as being contrary to public policy, the
As pointed out by the CA, WG&A and its related group of Court said:
companies, which were all extensively engaged in the shipping
Although in this jurisdiction, contracts of adhesion have been
business, had previously dry-docked and repaired its various
consistently upheld as valid per se; as binding as an ordinary
ships with KCSI under ship repair agreements incorporating the
contract, the Court recognizes instances when reliance on such
same standard conditions on at least 22 different occasions.
contracts cannot be favored especially where the facts and
Yet, in all these instances, WG&A had not been heard to
circumstances warrant that subject stipulations be
complain of being strong-armed and forced to accept the fine-
disregarded. Thus, in ruling on the validity and applicability of
print provisions imposed by KCSI to limit its liability.
the stipulation limiting the liability of CSEW for negligence to
Also, as pointed out by the CIAC, if it were true that WG&A did One Million (P1,000,000.00) Pesos only, the facts and
not want to be bound under such an onerous clause, it could circumstances vis-a-vis the nature of the provision sought to be
enforced should be considered, bearing in mind the than a year, Philamlife had not furnished Eternal with any reply
principles of equity and fair play. to the latter's insurance claim. This prompted Eternal to
demand from Philamlife the payment of the claim for
PhP100,000 on April 25, 1986. In response to Eternal's
demand, Philamlife denied Eternal's insurance claim in a letter
4. Eternal Gardens Memorial Park Corp. v. Philippine dated May 20, 1986, a portion of which reads:
American Life Insurance Co. The deceased was 59 years old when he entered into Contract
G.R. No. 166245, [April 8, 2008], 574 PHIL 161-174 #9558 and 9529 with Eternal Gardens Memorial Park in
October 1982 for the total maximum insurable amount of
P100,000.00 each. No application for Group Insurance was
DOCTRINE: An Insurance contract is a contract of adhesion submitted in our office prior to his death on August 2, 1984. In
which must be construed liberally in favor of the insured and accordance with our Creditor's Group Life Policy No. P-1920,
strictly against the insurer in order to safeguard the latter's under Evidence of Insurability provision, "a declaration of good
interest. health shall be required for all Lot Purchasers as party of the
application." We cite further the provision on Effective Date of
FACTS: Respondent Philippine American Life Insurance Coverage under the policy which states that "there shall be no
Company (Philamlife) entered into an agreement denominated insurance if the application is not approved by the Company."
as Creditor Group Life Policy No. P-1920 with Since no application had been submitted by the
petitioner Eternal Gardens Memorial Park Corporation Insured/Assured, prior to his death, for our approval but was
(Eternal). Under the policy, the clients of Eternal who purchased submitted instead on November 15, 1984, after his death, Mr.
burial lots from it on installment basis would be insured by John Uy Chuang was not covered under the Policy. We wish to
Philamlife. The amount of insurance coverage depended upon point out that Eternal Gardens being the Assured was a party to
the existing balance of the purchased burial lots. The policy was the Contract and was therefore aware of these pertinent
to be effective for a period of one year, renewable on a yearly provisions.
basis. Eternal was required under the policy to submit to
Philamlife a list of all new lot purchasers, together with a copy With regard to our acceptance of premiums, these do not
of the application of each purchaser, and the amounts of the connote our approval per se of the insurance coverage but are
respective unpaid balances of all insured lot purchasers. In held by us in trust for the payor until the prerequisites for
relation to the instant petition,Eternal complied by submitting a insurance coverage shall have been met. We will however,
letter dated December 29, 1982, containing a list of insurable return all the premiums which have been paid in behalf of John
balances of its lot buyers for October 1982. One of those Uy Chuang.
included in the list as "new business" was a certain John
Chuang. His balance of payments was PhP100,000. On August Consequently, Eternal filed a case before the Makati City
2, 1984, Chuang died. Regional Trial Court (RTC) for a sum of money against
Philamlife. The trial court decided in favor of Eternal. The RTC
Eternal sent a letter dated August 20, 1984 to Philamlife, which found that Eternal submitted Chuang's application for insurance
served as an insurance claim for Chuang's death. After more which he accomplished before his death, as testified to
by Eternal's witness and evidenced by the letter dated An examination of the above provision would show ambiguity
December 29, 1982, stating, among others: "Encl: Phil- between its two sentences. The first sentence appears to state
Am Life Insurance Application Forms & Cert." It further ruled that the insurance coverage of the clients of Eternal already
that due to Philamlife's inaction from the submission of the became effective upon contracting a loan with Eternal while the
requirements of the group insurance on December 29, 1982 to second sentence appears to require Philamlife to approve the
Chuang's death on August 2, 1984, as well as Philamlife's insurance contract before the same can become effective.
acceptance of the premiums during the same period, Philamlife
was deemed to have approved Chuang's application. The RTC It must be remembered that an insurance contract is a contract
said that since the contract is a group life insurance, once proof of adhesion which must be construed liberally in favor of the
of death is submitted, payment must follow. Philamlife appealed insured and strictly against the insurer in order to safeguard the
to the CA, which reversing the decision of the RTC.The CA latter's interest. Thus, in Malayan Insurance Corporation v.
based its Decision on the factual finding that Chuang's Court of Appeals, this Court held that:
application was not enclosed in Eternal's letter dated December Indemnity and liability insurance policies are
29, 1982. It further ruled that the non-accomplishment of the construed in accordance with the general rule of
submitted application form violated Section 26 of the Insurance resolving any ambiguity therein in favor of the
Code. Thus, the CA concluded, there being no application form, insured, where the contract or policy is prepared
Chuang was not covered by Philamlife's insurance. by the insurer. A contract of insurance, being
a contract of adhesion, par excellence, any
ambiguity therein should be resolved against
ISSUE: Whether Philamlife assumed the risk of loss without the insurer; in other words, it should be
approving the application. construed liberally in favor of the insured and
strictly against the insurer. Limitations of liability
should be regarded with extreme jealousy and
RULING: This question must be answered in the affirmative. must be construed in such a way as to preclude
As earlier stated, Philamlife and Eternal entered into an the insurer from noncompliance with its
agreement denominated as Creditor Group Life Policy No. P- obligations.
1920 dated December 10, 1980. In the policy, it is provided that:
In the more recent case of Philamcare Health Systems, Inc. v.
EFFECTIVE DATE OF BENEFIT. Court of Appeals, we reiterated the above ruling, stating that:
The insurance of any eligible Lot Purchaser shall When the terms of insurance contract contain
be effective on the date he contracts a loan with limitations on liability, courts should construe
the Assured. However, there shall be no them in such a way as to preclude the insurer
insurance if the application of the Lot Purchaser from non-compliance with his obligation. Being a
is not approved by the Company. contract of adhesion, the terms of an insurance
contract are to be construed strictly against the
party which prepared the contract, the insurer.
By reason of the exclusive control of the protect the interest of insurance applicants, insurance
insurance company over the terms and companies must be obligated to act with haste upon insurance
phraseology of the insurance contract, ambiguity applications, to either deny or approve the same, or otherwise
must be strictly interpreted against the insurer be bound to honor the application as a valid, binding, and
and liberally in favor of the insured, especially to effective insurance contract.
avoid forfeiture.
Clearly, the vague contractual provision, in Creditor 5. Philippine Health Care Providers, Inc. v. Commissioner
Group Life Policy No. P-1920 dated December 10, 1980, must of Internal Revenue
be construed in favor of the insured and in favor of the effectivity G.R. No. 167330 (Resolution), [September 18, 2009], 616
of the insurance contract. PHIL 387-423
On the other hand, the seemingly conflicting provisions must be
harmonized to mean that upon a party's purchase of a memorial DOCTRINE: PRINCIPAL OBJECT AND PURPOSE TEST is
lot on installment from Eternal, an insurance contract covering whether the assumption of risk and indemnification of loss
the lot purchaser is created and the same is effective, valid, and (which are elements of an insurance business) are the principal
binding until terminated by Philamlife by disapproving the object and purpose of the organization or whether they are
insurance application. The second sentence of Creditor merely incidental to its business. If these are the principal
Group Life Policy No. P-1920 on the Effective Date of Benefit is objectives, the business is that of insurance. But if they are
in the nature of a resolutory condition which would lead to the merely incidental and service is the principal purpose, then
cessation of the insurance contract. Moreover, the mere the business is not insurance.
inaction of the insurer on the insurance application must not
work to prejudice the insured; it cannot be interpreted as a
termination of the insurance contract. The termination of the FACTS: Petitioner is a domestic corporation whose primary
insurance contract by the insurer must be explicit and purpose is "[t]o establish, maintain, conduct and operate a
unambiguous. prepaid group practicehealth care delivery system or
As a final note, to characterize the insurer and the insured as a health maintenance organization to take care of the sick and
contracting parties on equal footing is inaccurate at best. disabled persons enrolled in the healthcare plan and to provide
Insurance contracts are wholly prepared by the insurer with vast for the administrative, legal, and financial responsibilities of the
amounts of experience in the industry purposefully used to its organization". Individuals enrolled in its healthcare programs
advantage. More often than not, insurance contracts are pay an annual membership fee and are entitled to various
contracts of adhesion containing technical terms and conditions preventive, diagnostic and curative medical services provided
of the industry, confusing if at all understandable to laypersons, by its duly licensed physicians, specialists and other
that are imposed on those who wish to avail of insurance. As professional technical staff participating in the group
such, insurance contracts are imbued with public interest that practice health delivery system at a hospital or clinic owned,
must be considered whenever the rights and obligations of the operated or accredited by it.
insurer and the insured are to be delineated. Hence, in order to
On January 27, 2000, respondent Commissioner of Internal not as merely incidental to any other
Revenue [CIR] sent petitioner a formal demand letter and the legitimate business or activity of the
corresponding assessment notices demanding the payment of surety;
deficiency taxes, including surcharges and interest, for the
taxable years 1996 and 1997 in the total amount of c) doing any kind of business, including a
P224,702,641.18.The deficiency [documentary stamp tax reinsurance business, specifically
(DST)] assessment was imposed on recognized as constituting the doing of
petitioner's health care agreement with the members of its an insurance business within the
health care program pursuant to Section 185 of the 1997 Tax meaning of this Code;
Code. Petitioner protested the assessment in a letter dated d) doing or proposing to do any business in
February 23, 2000. As respondent did not act on the protest, substance equivalent to any of the
petitioner filed a petition for review in the Court of Tax Appeals foregoing in a manner designed to evade
(CTA) seeking the cancellation of the deficiency VAT and DST the provisions of this Code.
assessments.
In the application of the provisions of this Code,
On April 5, 2002, the CTA rendered a decision partially granting the fact that no profit is derived from the making
the petition. Respondent appealed the CTA decision to the of insurance contracts, agreements or
[Court of Appeals (CA)] insofar as it cancelled the DST transactions or that no separate or direct
assessment. He claimed that consideration is received therefore, shall not be
petitioner's health care agreement was a contract of insurance deemed conclusive to show that the making
subject to DST under Section 185 of the 1997 Tax Code. On thereof does not constitute the doing or
August 16, 2004, the CA rendered its decision. It held that transacting of an insurance business.
petitioner's health care agreement was in the nature of a non-
life insurance contract subject to DST. Various courts in the United States, whose jurisprudence has a
persuasive effect on our decisions, have determined that
HMOs are not in the insurance business. One test that they
ISSUE: Whether the petitioner, as a health maintenance have applied is whether the assumption of risk and
organization (HMO), engaged in the insurance business. indemnification of loss (which are elements of an insurance
business) are the principal object and purpose of the
RULING: NO. Section 2 (2) of PD 1460 (otherwise known as organization or whether they are merely incidental to its
the Insurance Code) enumerates what constitutes "doing an business. If these are the principal objectives, the business is
insurance business" or "transacting an insurance business": that of insurance. But if they are merely incidental and service
is the principal purpose, then the business is not insurance.
a) making or proposing to make, as insurer, any
insurance contract; Applying the "principal objects and purpose test", a corporation
(such as an HMO, whether or not organized for profit), whose
b) making or proposing to make, as surety, any main object is to provide the members of a group with health
contract of suretyship as a vocation and
services, is not engaged in the insurance business. rather than insurance services, it cannot be considered as being
onsequently, the mere presence of risk would be insufficient to in the insurance business.
override the primary purpose of the business to provide medical
It is important to emphasize that, in adopting the "principal
services as needed, with payment made directly to the provider
purpose test" , for the purpose of determining what "doing an
of these services. In short, even if petitioner assumes the risk of
insurance business" means, we have to scrutinize the
paying the cost of these services even if significantly more than
operations of the business as a whole and not its mere
what the member has prepaid, it nevertheless cannot be
components. This is of course only prudent and appropriate,
considered as being engaged in the insurance business.
taking into account the burdensome and strict laws, rules and
By the same token, any indemnification resulting from the regulations applicable to insurers and other entities engaged in
payment for services rendered in case of emergency by non- the insurance business.
participating health providers would still be incidental to
petitioner's purpose of providing and arranging
for health care services and does not transform it into an 6. Enriquez v. Sun Life Assurance Company of Canada
insurer. To fulfill its obligations to its members under the G.R. No. 15895, [November 29, 1920], 41 PHIL 269-275
agreements, petitioner is required to set up a system and the
facilities for the delivery of such medical services. This DOCTRINE:
indubitably shows that indemnification is not its sole object.  The Civil Code rule, that an acceptance made by letter
shall bind the person making the offer only from the date
In fact, a substantial portion of petitioner's services covers
it came to his knowledge, is controlling. The contract for
preventive and diagnostic medical services intended to keep
a life annuity was not perfected because it had not been
members from developing medical conditions or diseases. As
proved satisfactorily that the acceptance of the
an HMO, it is its obligation to maintain the good health of its
application ever came to the knowledge of the applicant.
members. Accordingly, its health care programs are
designed to prevent or to minimize the possibility of any  An acceptance of an offer of insurance not actually or
assumption of risk on its part. Thus, its undertaking under its constructively communicated to the proposer does not
agreements is not to indemnify its members against any loss or make a contract. Only the mailing of acceptance
damage arising from a medical condition but, on the contrary, completes the contract of insurance, as the locus
to provide the health and medical services needed to prevent poenitentiae is ended when the acceptance has passed
such loss or damage. beyond the control of the party.|||

Overall, petitioner appears to provide insurance-type benefits to FACTS: On September 24, 1917, Joaquin Herrer made
its members (with respect to its curative medical services), but application to the Sun Life Assurance Company of Canada
these are incidental to the principal activity of providing them through its office in Manila for a life annuity. Two days later he
medical care. The "insurance-like" aspect of petitioner's paid the sum of P6,000 to the manager of the company's Manila
business is miniscule compared to its non-insurance activities. office and was given a receipt reading as follows:
Therefore, since it substantially provides health care services
"MANILA, I. F., 26 de
septiembre, 1917.
"PROVISIONAL RECEIPT noticed that it is identical with the principles announced by a
"P6,000 considerable number of respectable, courts in the United
States. The courts who take this view have expressly held that
"Recibi la suma de seis mil pesos de an acceptance of an offer of insurance not actually or
Don Joaquin-Herrer de Manila como prima de constructively communicated to the proposer does not make a
la Renta Vitalicia solicitada por dicho Don contract. Only the mailing of acceptance, it has been said,
Joaquin Herrer hoy, sujeta al examen medico completes the contract of insurance, as the locus
y aprobacion de la Oficina Central de la poienitentise is ended when the acceptance has passed
Compañia." beyond the control of the party. (I Joyce, The Law of Insurance,
The application was immediately forwarded to the head office pp. 235, 244.)
of the company at Montreal, Canada. On November 26, 1917, In resume, therefore, the law applicable to the case is found to
the head office gave notice of acceptance by cable to Manila. be the second paragraph of! article 1262 of the Civil Code
(Whether on the same day the cable was received notice was providing that an acceptance made by letter shall not bind the
sent by the Manila office to Herrer that the application had been person making the offer except from the time it came to his
accepted, is a disputed point, which will be discussed later.) On knowledge. The pertinent fact is, that according to the
December 4, 1917, the policy was issued at Montreal. On provisional receipt, three things had to be accomplished by the
December 18, 1917, attorney Aurelio A. Torres wrote to the insurance company before there was a contract: (1) There had
Manila office of the company stating that Herrer desired to to be a medical examination of the applicant; (2) there had to be
withdraw his application. The following day the local office approval of the application by the head office of the company;
replied to Mr. Torres, stating that the policy had been issued, and (3) this approval had in some way to be communicated by
and called attention to the notification of November 26, 1917. the company to the applicant. The further admitted facts are that
This letter was received by Mr. Torres on the morning of the head office in Montreal did accept the application, did cable
December 21, 1917. Mr. Herrer died on December 20, 1917. the Manila office to that effect, did actually issue the policy and
ISSUE: Whether the contract for a life annuity in the case at bar did, through its agent in Manila, actually write the letter of
was perfected. notification and place it in the usual channels for transmission
to the addressee. The fact as to the letter of notification thus
RULING: NO. We hold that the contract for a life annuity in the fails to concur with the essential elements of the general rule
case at bar was not perfected because it has not been proved pertaining to the mailing and delivery of mail matter as
satisfactorily that the acceptance of the application ever came announced by the American courts, namely, when a letter or
to the knowledge of the applicant. other mail matter is addressed and mailed with postage prepaid
The Civil Code rule, that an acceptance made by letter shall there is a rebuttable presumption of fact that it was received by
bind the person making the offer only from the date it came to the addressee as soon as it could have been transmitted to him
his knowledge, may not be the best expression of modern in the ordinary course of the mails. But if any one of these
commercial usage. Still it must be admitted that its enforcement elemental facts fails to appear, it is fatal to the presumption. For
avoids uncertainty and tends to security. Not only this, but in instance, a letter will not be presumed to have been received by
order that the principle may not be taken too lightly, let it be
the addressee unless it is shown that it was deposited in the Issue
post-office, properly addressed and stamped.
Whether or not the common-law-wife named as beneficiary in
the life insurance policy of a legally married man can claim the
proceeds thereof.

7. Insular Life Assurance Company vs Ebrado Held

GR No. L-44059 October 28,1977 No. Article 2011 of the New Civil Code states that "the contract
of insurance is governed by special laws. Matters not
80 SCRA 181 expressly provided for in such special laws shall be regulated
Doctrine by this Code." When not otherwise specifically provided for by
the Insurance Law, the contract of life insurance is governed
Under Art. 2012 of the NCC, "any person who is forbidden by the general rules of the civil law regulating contracts. And
from receiving any donation under Art. 739 cannot be named under Art. 2012 of the NCC, "any person who is forbidden from
beneficiary of a life insurance policy by the person who cannot receiving any donation under Art. 739 cannot be named
make a donation to him." Common-law spouses are definitely beneficiary of a life insurance policy by the person who cannot
barred from receiving donations from each other. make a donation to him." Common-law spouses are definitely
barred from receiving donations from each other.
Facts
The reason is because in essence, a life insurance policy is no
Buenaventura Ebrado was issued by The Life Assurance Co
different from a civil donation insofar as the beneficiary is
an insurance policy and designated respondent Carpiona
concerned. Both are founded upon the same consideration:
Ebrado, his wife, as the revocable beneficiary. Buenaventura
liberality. A beneficiary is like a donee, because from the
died. Carpiona filed with the insurer a claim for the proceeds of
premiums of the policy which the insured pays out of liberality,
the Policy as the designated beneficiary therein, although she
the beneficiary will receive the proceeds of said insurance. As
admits that she and the insured were merely living as husband
a consequence, the proscription in Art. 739 of the NCC should
and wife without the benefit of marriage. Pascuala Vda. de
equally operate in life insurance contracts.
Ebrado also filed her claim as the widow of the insured. She
asserts that she is the one entitled to the proceeds, not
Carpiona. In doubt, petitioner commenced an action for
interpleader. The RTC declared Carpiona is disqualified from
becoming beneficiary and ordered the payment of the
proceeds to the estated of the insured. Upon appeal, the CA
directed the case to SC.
8. Heirs of Loreto Maramag vs Maramag and divided the proceeds among the illegitimate children as
the remaining designated beneficiaries pursuant to Sec. 53 of
GR No. 181132 June 5, 2009
the Insurance Code.
588 SCRA 744
The RTC ruled in favor of petitioners but thereafter set aside
Doctrine its former decision in the MR. The RTC ruled that pursuant to
Sec. 53 of the Insurance Code the entire proceeds should be
Sec. 53 of the Insurance Code provides that the insurance paid to the illegitimate children of Loreto, considering that Eva
proceeds shall be applied exclusively to the proper interest of the common-law wife was disqualified. The CA affirmed the
the person in whose name or for whose benefit it was made RTC.
unless otherwise provided in the policy. Pursuant thereto, it is
obvious that the only persons entitled to claim the insurance Issue
proceeds are either the insured, if still alive, or the beneficiary,
Whether or not the petitioners can claim the insurance
if the insured is already deceased, upon the maturity of the
proceeds despite not being the named beneficiaries.
policy.
Held
Facts:
No. The petitioners prays that the share of Eva, being
Petitioners filed an action for revocation and/or reduction of the
disqualified as beneficiary, and portions of the share of
insurance proceeds in favor of respondents for being void
Loreto's illegitimate children should be awarded to them.
and/or inofficious. The petition alleged that: (1) petitioners
However, in the case, it is clear from the petition that although
were the legitimate wife and children of Loreto Maramag, while
petitioners are the legitimate heirs of Loreto, they were not
respondents were Loreto's illegitimate family; (2) Respondents
named as beneficiaries in the insurance policies issued by
Eva Maramag was a concubine of and a suspect in the killing
Insular and Grepalife. Sec. 53 of the Insurance Code provides
of Loreto, thus she is disqualified to receive any proceeds from
that the insurance proceeds shall be applied exclusively to the
his insurance policies from Insular Life and Grepalife; (3) the
proper interest of the person in whose name or for whose
illegitimate children of Loreto - respondents Odessa, Karl
benefit it was made unless otherwise provided in the policy.
Brian, and those to be released to Karl Brian and Trisha
Pursuant thereto, it is obvious that the only persons entitled to
Angelie were inofficious and should be reduced; and (4)
claim the insurance proceeds are either the insured, if still
petitioners could not be deprived of their legitimes, which
alive, or the beneficiary, if the insured is already deceased,
should be satisfied first. ( in short, the petitioners were
upon the maturity of the policy.
claiming that they should be entitled to the proceeds or atleast
a part thereof, and not the respondents who were designated Petitioners are third parties to the insurance contracts with
as beneficiaries ) Insular answered that when it ascertained Insular and Grepalife and thus are not entitled to the proceeds
Eva was not the legal wife, it disqualified her as a beneficiary thereof. Accordingly, Insular and Grepalife have no obligation
to turn over the proceeds to petitioners. The revocation of Eva
as beneficiary and her disqualification in another are of no
moment considering that the designation of the illegitimate
children as beneficiaries remains valid. Because no legal
proscription exists in naming as beneficiaries the children of
illicit relationships by the insured, the shares of Eva in the
insurance proceeds must be awarded to the designated
illegitimate children to the exclusion of petitioners
9. Gercio vs Sun Life Assurance of Canada Whether or not the insured husband has the power to change
the beneficiary - the former wife - and to name instead his
GR No. 23703 September 28, 1925
actual wife, where the insured and the beneficiary have been
48 PHIL 53 divorced and where the policy does not expressly reserve to
the insured the right to change the beneficiary.
Doctrine
Held
As soon as the policy was issued, the wife acquired a vested
interest therein, of which she could not be deprived without her No. The Court cited American cases similar to the case at bar.
consent, except under the terms of the contract with the The Court cited Filley vs Illinois Life Insurance. Under the
insurance company. It remains her separate property after the case, the Court ruled that the benefit accruing from a policy of
divorce as before. life insurance upon the life of a married man, payable upon the
death to his wife, naming her, is payable to the surviving
Facts beneficiary named, although she may have years thereafter
Respondent Sun Life issued an insurance policy known as a secured a divorce from her husband, and hewas thereafter
twenty-year endowment policy on the life of petitioner Gercio. again married to one who sustained the relation of wife to him
The policy designated Mrs. Andrea Zalcita, his wife, as at the time of his death. The rights of a beneficiary in an
beneficiary should she survive him, otherwise to the executors, ordinary life insurance policy become vested upon the
administrators or assigns of the insured. The policy did not issuance of the policy, and can thereafter, during the life of the
include any provision reserving to the insured the right to beneficiary, be defeated only as provided by the terms of the
change the beneficiary. policy.

On the date of the policy Zialcita was the lawful wife of Gercio. The Court also cited Wallace vs Mutual Benefit Life
Thereafter, Zialcita was convicted of adultey. A divorce decree Assurance. Under the case, as soon as the policy was issued,
was later on issued dissolving the bonds of Gercio and the wife acquired a vested interest therein, of which she could
Zialcita. On March 1922, Gercio formally notified respondent not be deprived without her consent, except under the terms of
that he had revoked his donation in favor of Zialcita and that the contract with the insurance company. No right to change
he had designated in her stead his present wife, Adela de the beneficiary was reserved. Her interest in the policy was her
Gercio, as the beneficiary of the policy. Gercio requested the individual policy, subject to be divested only by her death, the
insurance company to eliminate Zialcita as beneficiary but the lapse of time, or by failure to pay the premiums. The wife's
responted refused to do so. Gercio thus filed a complaint for interest was not affected by the decree of court which
mandamus. The RTC ruled in favor of Gercio ordering the dissolves the marriage contract between the parties. It remains
elimination of the designation of Zialcita. her separate property after the divorce as before.

Issue
10. Go vs. Redfern Held:
When designated in a policy, the beneficiary acquires a right of
NOTE: Spanish yung full text ng case @_@ although
which he cannot be deprived of without his consent, unless the
nakahanap ako ng translated digest.
right has been reserved specifically to the insured to modify
GR 47705, 25 April 1941 the policy. The same doctrine was enunciated by the Court in
the cases of Gercio vs. Sun Life Assurance Co. of Canada and
72 PHIL 25 Insular Life vs. Suva Thus, unless the insured has reserved
Doctrine specifically the right to change or to modify the policy, with
respect to the beneficiary, said policy constitutes an acquired
When designated in a policy, the beneficiary acquires a right of right of the beneficiary, which cannot be modified except with
which he cannot be deprived of without his consent, unless the the consent of the latter. Herein, it is admitted that Redfern did
right has been reserved specifically to the insured to modify not reserve expressly his right to change or modify the policy.
the policy Change implies the idea of an alteration. The addition of Go's
name as one of the beneficiaries of the policy constitutes
Facts: change as all addition is an alteration. The addition of Go's
In October 1937, Edward K. Redfern obtained an insurance name changed the policy inasmuch as there are two
policy against accidents from the International Assurance Co, beneficiaries instead of one, and thus in effect the original
Ltd. On 31 August 1938, Redfern died from an accident. The beneficiary cannot receive the full amount of the policy. The
mother of the deceased, presenting the necessary evidence of Supreme Court affirmed the appealed judgment in all of its
the death of Redfern, sought to claim the proceeds of the parts, with costs against Go.
insurance policy from the insurance company. The company,
however, denied such claim, on the ground that the insurance
policy was amended on 22 November 1937 to include another
beneficiary, Concordia Go. Hence, an action was filed to
determine who has the right to collect the insurance proceeds
of the deceased Redfern. The mother claimed that the addition
of the co-beneficiary is illegal. Go, on her part, alleged the
contrary. The trial court ruled in favor of Angela Redfern, the
mother. Go appealed.

Issue:
Whether the addition of Go’s name as co-beneficiary can be
allowed for her share in the insurance proceeds
11. Gaisano Cagayan Inc vs Insurance Company of North their claim under their respective fire insurance policies with
America book debt endorsements, and after payment thus there is
subrogation; and that respondent seeks to claim the unpaid
accounts of petitioner on the sale and delivery of ready-made
GR No. 147839 June 8, 2006 clothing materials. Petitioner contend in its Answer that it could
not be held liable because the property covered by the policies
Gaisano Cagayan Inc vs Insurance Company of North were destroyed by force majeure. The RTC dismissed the
America complaint on the ground that loss was due to force majeure.
490 SCRA 286 The CA reversed the RTC. The CA ruled that the fire
insurance policy on book debts covers the unpaid accounts of
Doctrine IMC and LSPI since such insurance applies to loss of the
ready-made clothing materials sold and delivered to petitioner.
Unlike the civil law concept of res perit domino, where
ownership is the basis for consideration of who bears the loss, Issue
in property insurance, one's interest is not determined by
concept of title, but whether insured has substantial economic Whether or not petitioner is liable to respondent for the unpaid
interest in the property. account

Facts Held

Intercapitol Marketing Corporation (IMC) and Levi Strauss Yes. It is well-settled that when the words of a contract are
Philippines (LSPI) separately obtained from respondent fire plain and readily understood, there is no room for construction.
insurance policies with book debt endorsements. The Nowhere is it provided in the questioned policies that the
insurance policies provide for coverage on "book debts in subject of the insurance is the goods sold and delivered to the
connection with ready-made clothing materials which have customers and dealers of the insured. The policies clearly
been sold or delivered to various customers and dealers of the proved coverage for Book Debts of the insured.
insured anywhere in the Philippines. Under the policy, book IMC and LSPI did not lose complete interest over the goods.
debt is defined as " unpaid account still appearing in the Book They have an insurable interest until full payment of the value
of Account of the Insured 45 days after the time of the loss." of the delivered goods. Unlike the civil law concept of res perit
Petitioner is a customer and dealer of the products of IMC and domino, where ownership is the basis for consideration of who
LSPI. On February 1991, Gaisano Superstore Complex, bears the loss, in property insurance, one's interest is not
owned by petitioner, was consumed by fire burning the stocks determined by concept of title, but whether insured has
of the clothing material sold and delivered by IMC and LSPI. substantial economic interest in the property. Therefore, an
Respondent filed a complaint for damages against petitioner. insurance interest in property does not necessarily imply a
The complaint alleges that IMC and LSPI filed with respondent property interest in, or a lien upon, or possession of the
subject matter of insurance, and neither title nor a beneficial
interest is requisite to the existence of such an interest. It is
sufficient that the insured is so situated with reference to the
propery that he would be liable to the loss should it be injured
or destroyed by the peril against which it is insured.
Indeed, a vendor retains an insurable interest in the property
sold so long as he has any interest therein, In other words, as
long as he would suffer by its destruction, as where he has a
vendor's lien. In this case the insurable interest of IMC and
LSPI pertain to the unpaid accounts appearing in their Book of
Account 45 days after the time of the loss covered by the
policies.
12. RCBC vs CA payment of the fire loss claims. The CA affirm and ordered to
release the amount of the attached policy in favor of GOYU.
GR No. 128833 April 20, 1998
Both Courts rule that GOYU (not RCBC) is entitled to the
289 SCRA 292
proceeds although the policies were endorsed to RCBC
Doctrine because the endorsements made by Alchester do not bear the
signature of any officer of GOYU hence such were defective.
It is settled that a mortgagor and mortgagee have separate
and distinct insurable interest in the same mortgaged property, Issue
such that each one of them may insure the same property for
Whether or not GOYU is entitled to the proceeds of the fire
his own sole benefit.
insurance policy.
Facts
Held
Respondent Goyu & Sons (GOYU) applied for credit facilities
No. It is settled that a mortgagor and mortgagee have
and accomodations with RCBC which where thereafter
separate and distinct insurable interest in the same mortgaged
granted. As security for its credit facilities with RCBC, GOYU
property, such that each one of them may insure the same
executed 2 Real Estate Mortgages and 2 Chattel Mortgages in
property for his own sole benefit.
favor of RCBC. Under the 4 mortgage contracts, GOYU
committed to insure the mortgaged properties with an Although it appears that GOYU obtained the subject
insurance company approved by RCBC, and subsequently to insurance policies naming itself as the sole payee, the
endorse and deliver the policies to RCBC. GOYU obtained a intention of the parties as shown by their contemporaneous
total of 10 insurance policies from Malayan Insurance acts, must be given due consideration. It is to be noted that 9
Company (MICO). Alchester Insurance, the insurance agent endorsement documents were prepared by Alchester in favor
where GOYU obtained the policies, issued 9 endorsements in of RCBC. RCBC, in good faith, relied upon the endorsement
favor of RCBC upon instructions of GOYU. On April 1992, one documents sent to it as this was only pursuant to the
of GOYU's factory building was gutted by fire. GOYU stipulation in the mortgage contracts. GOYU cannot seek relief
submitted its claim for indemnity against MICO. MICO denied under Sec. 53 of the Insurance Code which provides that the
on the ground that the policies were attached pursuant to writs proceeds of insurance shall exclusively apply to the interest of
of attachments/garnishments issued by various courts by other the person in whose name or for whose benefit it is made. The
creditors of GOYU. RCBC, one of the creditors of GOYU, also peculiarity of the circumstances obtaining in the instant case
filed a claim with MICO but were denied on the same grounds. presents a justification to take exception to the strict
application of Sec. 53, it having been established that it was
GOYU filed a claim for specific performance against MICO.
the intention of the parties to designate RCBC as the party for
The RTC ruled in favor of GOYU ordering MICO and RCBC
whose benefit the insurance policies were taken out. The
intention of the parties will have to be given full force and On the other hand, a mortgagee may also procure a policy as a
effect. The insurance proceeds may, therefore, be exclusively contracting party in accordance with the terms of an agreement
applied to RCBC, which under the factual circumstances of the by which the mortgagor is to pay the premiums upon such
case is truly the entity for whose benefit the policies were insurance. It has been noted, however, that although the
clearly intended. mortgagee is himself the insured, as where he applies for a
policy, fully informs the authorized agent of his interest, pays the
13. GEAGONIA v. CA (COUNTRY BANKERS INSURANCE)
premiums, and obtains on the assurance that it insures him, the
DAVIDE; February 6 1995 policy is in fact in the form used to insure a mortgagor with loss
payable clause.
DOCTRINE:
FACTS:
A mortgagor may, however, take out insurance for the benefit
of the mortgagee, which is the usual practice. The mortgagee Geagonia is the owner of Norman's Mart located in the public
may be made the beneficial payee in several ways. He may market of San Francisco, Agusan del Sur. On 22 Dec 1989, he
become the assignee of the policy with the consent of the obtained from the private respondent fire insurance policy for
insurer; or the mere pledgee without such consent; or the P100,000.00. The period of the policy was from 22 Dec 1989 to
original policy may contain a mortgage clause; or a rider making 22 Dec 1990 and covered the ff: "Stock-in-trade consisting
the policy payable to the mortgagee "as his interest may principally of dry goods such as RTW's for men and women
appear" may be attached; or a "standard mortgage clause," wear and other usual to assured's business.
containing a collateral independent contract between the
The policy contained the following condition:
mortgagee and insurer, may be attached; or the policy, though
by its terms payable absolutely to the mortgagor, may have "3. The insured shall give notice to the Company of any
been procured by a mortgagor under a contract duty to insure insurance or insurances already effected, or which may
for the mortgagee's benefit, in which case the mortgagee subsequently be effected, covering any of the property or
acquires an equitable lien upon the proceeds. properties consisting of stocks in trade, goods in process and/or
inventories only hereby insured, and unless notice be given and
In the policy obtained by the mortgagor with loss payable clause
the particulars of such insurance or insurances be stated therein
in favor of the mortgagee as his interest may appear, the
or endorsed in this policy pursuant to Section 50 of the
mortgagee is only a beneficiary under the contract, and
Insurance Code, by or on behalf of the Company before the
recognized as such by the insurer but not made a party to the
occurrence of any loss or damage, all benefits under this policy
contract himself. Hence, any act of the mortgagor which defeats
shall be deemed forfeited, provided however, that this condition
his right will also defeat the right of the mortgagee. This kind of
shall not apply when the total insurance or insurances in force
policy covers only such interest as the mortgagee has at the
at the time of the loss or damage is not more than P200,000.00."
issuing of the policy.
On 27 May 1990, fire of accidental origin broke out at around claimed under the three policies was below the actual value of
7:30 p.m. at the public market of San Francisco, Agusan del his stocks at the time of loss, w/c was P1M.
Sur. The petitioner's insured stocks-in-trade were completely
The Insurance Commission found that the petitioner did not
destroyed prompting him to file w/ the private respondent a
violate Condition 3 as he had no knowledge of the existence of
claim under the policy. On 28 Dec 1990, the private respondent
the two fire insurance policies obtained from the PFIC; that it
denied the claim because it found that at the time of the loss the
was Cebu Tesing Textiles w/c procured the PFIC policies w/o
petitioner's stocks-in-trade were likewise covered by two fire
informing him or securing his consent; and that Cebu Tesing
insurance policies for P100,000.00 each, issued by the Cebu
Textile, as his creditor, had insurable interest on the stocks.
Branch of the Philippines First Insurance Co., Inc. (PFIC).
These findings were based on the petitioner's testimony that he
These policies indicate that the insured was "Messrs. Discount
came to know of the PFIC policies only when he filed his claim
Mart (Mr. Armando Geagonia, Prop.)" with a mortgage clause
with the private respondent and that Cebu Tesing Textile
reading:
obtained them and paid for their premiums w/o informing him.
MORTGAGE: Loss, if any shall be payable to Messrs. Cebu The Insurance Commission then ordered the respondent
Tesing Textiles, Cebu City as their interest may appear subject company to pay complainant the sum of P100,000.00 with legal
to the terms of this policy. CO-INSURANCE DECLARED: interest from the time the complaint was filed until fully satisfied
P100,000. — Phils. First CEB/F 24758 plus the amount of P10,000.00 as attorney's fees.
CA reversed the decision of the Insurance Commission
because it found that the petitioner knew of the existence of the
The basis of the private respondent's denial was the petitioner's
two other policies issued by the PFIC.
alleged violation of Condition 3 of the policy.
Geagonia then filed a complaint against the private respondent
w/ the Insurance Commission for the recovery of P100,000.00 ISSUES
under fire insurance policy, for attorney's fees, and costs of
WON the petitioner had prior knowledge of the two insurance
litigation. He claims that the time he obtained the private
policies issued by the PFIC when he obtained the fire insurance
respondent's fire insurance policy he knew that the two policies
policy from the private respondent, thereby, for not disclosing
issued by the PFIC were already in existence; however, he had
such fact, violating Condition 3 of the policy which would
no knowledge of the provision in the private respondent's policy
preclude him from recovering therefrom.
requiring him to inform it of the prior policies; this requirement
was not mentioned to him by the private respondent's agent; HELD:
and had it been so mentioned, he would not have withheld such NO.
information. He further asserted that the total of the amounts
Condition 3 of the private respondent's Policy No. F-14622 is a Loss, if any, shall be payable to MESSRS. TESING TEXTILES,
condition which is not proscribed by law. Its incorporation in the Cebu City as their interest may appear subject to the terms of
policy is allowed by Section 75 of the Insurance Code which this policy.
provides that "[a] policy may declare that a violation of specified
This is clearly a simple loss payable clause, not a standard
provisions thereof shall avoid it, otherwise the breach of an
mortgage clause.
immaterial provision does not avoid the policy." Such a
condition is a provision which invariably appears in fire Provisions, conditions or exceptions in policies which tend to
insurance policies and is intended to prevent an increase in the work a forfeiture of insurance policies should be construed most
moral hazard. It is commonly known as the additional or "other strictly against those for whose benefits they are inserted, and
insurance" clause and has been upheld as valid and as a most favorably toward those against whom they are intended to
warranty that no other insurance exists. Its violation would thus operate. The reason for this is that, except for riders which may
avoid the later be inserted, the insured sees the contract already in its
policy. However, in order to constitute a violation, the other final form and has had no voice in the selection or arrangement
insurance must be upon same subject matter, the same interest of the words employed therein. On the other hand, the language
therein, and the same risk. of the contract was carefully chosen and deliberated upon by
experts and legal advisers who had acted exclusively in the
As to a mortgaged property, the mortgagor and the mortgagee
interest of the insurers and the technical language employed
have each an independent insurable interest therein and both
therein is rarely understood by ordinary laymen.
interests may be one policy, or each may take out a separate
policy covering his interest, either at the same or at separate With these principles in mind, we are of the opinion that
times. The mortgagor's insurable interest covers the full value Condition 3 of the subject policy is not totally free from ambiguity
of the mortgaged property, even though the mortgage debt is and must, perforce, be meticulously analyzed. Such analysis
equivalent to the full value of the property. The mortgagee's leads us to conclude that (a) the prohibition applies only to
insurable interest is to the extent of the debt, since the property double insurance, and (b) the nullity of the policy shall only be
is relied upon as security thereof, and in insuring he is not to the extent exceeding P200,000.00 of the total policies
insuring the property but his interest or lien thereon. His obtained.
insurable interest is prima facie the value mortgaged and
extends only to the amount of the debt, not exceeding the value The first conclusion is supported by the portion of the condition
of the mortgaged property. Thus, separate insurances covering referring to other insurance "covering any of the property or
different insurable interests may be obtained by the mortgagor properties consisting of stocks in trade, goods in process and/or
and the mortgagee. inventories only hereby insured," and the portion regarding the
insured's declaration on the subheading CO-INSURANCE that
The fire insurance policies issued by the PFIC name the the co-insurer is Mercantile Insurance Co., Inc. in the sum of
petitioner as the assured and contain a mortgage clause which P50,000.00. A double insurance exists where the same person
reads: is insured by several insurers separately in respect of the same
subject and interest. As earlier stated, the insurable interests of
a mortgagor and a mortgagee on the mortgaged property are
14. SOUTH SEA SURETY AND INSURANCE COMPANY,
distinct and separate. Since the two policies of the PFIC do not
INC., petitioner, vs. HON. COURT OF APPEALS and
cover the same interest as that covered by the policy of the
VALENZUELA HARDWOOD AND INDUSTRIAL SUPPLY,
private respondent, no double insurance exists. The non-
INC., respondents
disclosure then of the former policies was not fatal to the
petitioner's right to recover on the private respondent's policy.
Furthermore, by stating within Condition 3 itself that such 244 SCRA 744 June 2, 1995
condition shall not apply if the total insurance in force at the time
of loss does not exceed P200,000.00, the private respondent VITUG
was amenable to assume a co-insurer's liability up to a loss not
exceeding P200,000.00. What it had in mind was to discourage
over-insurance. Indeed, the rationale behind the incorporation DOCTRINE:
of "other insurance" clause in fire policies is to prevent over-
Undoubtedly, the payment of the premium is a condition
insurance and thus avert the perpetration of fraud. When a
precedent to, and essential for, the efficaciousness of the
property owner obtains insurance policies from two or more
contract. The only two statutorily provided exceptions are (a) in
insurers in a total amount that exceeds the property's value, the
case the insurance coverage relates to life or industrial life
insured may have an inducement to destroy the property for the
(health) insurance when a grace period applies and (b) when
purpose of collecting the insurance. The public as well as the
the insurer makes a written acknowledgment of the receipt of
insurer is interested in preventing a situation in which a fire
premium, this acknowledgment being declared by law to be
would be profitable to the insured.
then conclusive evidence of the premium payment.
FACTS:
On 16 January 1984, plaintiff [Valenzuela Hardwood and
Industrial Supply, Inc.] entered into an agreement with the
defendant Seven Brothers whereby the latter undertook to load
on board its vessels M/V Seven Ambassador the former's lauan
round logs numbering 940 at the port of Maconacon, Isabela for
shipment to Manila.
On 20 January 1984, plaintiff insured the logs against loss
and/or, damage with defendant South Sea Surety and
Insurance Co., Inc. for P2,000,000.00 and the latter issued its
Marine Cargo Insurance Policy No. 84/24229 for P2,000,000.00 HELD:
on said date. The plaintiff gave the check in payment of the
YES.
premium on the insurance policy to Mr. Victorio Chua.
No attempt to becloud the issues can disguise the fact that the
In the meantime, the said vessel M/V Seven Ambassador sank
sole question raised in the instant petition is really evidentiary in
on 25 January 1984 resulting in the loss of the plaintiff's insured
nature, i.e., whether or not Victorio Chua, in receiving the check
logs. On 30 January 1984, a check for P5,625.00 (Exh. 'E') to
for the insurance premium prior to the occurrence of the risk
cover payment of the premium and documentary stamps due
insured against has so acted as an agent of petitioner. The
on the policy was tendered to the insurer but was not accepted.
appellate court, like the trial court, has found in the affirmative.
Instead, the South Sea Surety and Insurance Co., Inc.
Said the appellate court: "In the instant case, the Marine Cargo
cancelled the insurance policy it issued as of the date of
Insurance Policy No. 84/24229 was issued by defendant
inception for non-payment of the premium due in accordance
insurance company on 20 January 1984. At the time the vessel
with Section 77 of the Insurance Code.
sank on 25 January 1984 resulting in the loss of the insured
On 2 February 1984, plaintiff demanded from defendant South logs, the insured had already delivered to Victorino Chua the
Sea Surety and Insurance Co., Inc. the payment of the check in payment of premium. But, as Victorino Chua testified,
proceeds of the policy but the latter denied liability under the it was only in the morning of 30 January 1984 or 5 days after
policy. Plaintiff likewise filed a formal claim with defendant the vessel sank when his messenger tendered the check to
Seven Brothers Shipping Corporation for the value of the lost defendant South Sea Surety and Insurance Co., Inc. "The
logs but the latter denied the claim. pivotal issue to be resolved to determine the liability of the
surety corporation is whether Mr. Chua acted as an agent of the
the trial court rendered judgment in favor of plaintiff Hardwood.
surety company or of the insured when he received the check
On appeal perfected by both the shipping firm and the insurance for insurance premiums. "Appellant surety company insists that
company, the Court of Appeals affirmed the judgment of the Mr. Chua is an administrative assistant for the past ten years
court a quo only against the insurance corporation; and an agent for less than ten years of the Columbia Insurance
Brokers, Ltd. He is paid a salary as administrative assistant and
In the Court, petitioner argues that it likewise should have been a commission as agent based on the premiums he turns over to
freed from any liability to Hardwood. It faults the appellate court the broker. Appellant therefore argues that Mr. Chua, having
(a) for having supposedly disregarded Section 77 of the received the insurance premiums as an agent of the Columbia
Insurance Code and (b) for holding Victorio Chua to have been Insurance Broker, acted as an agent of the insured under
an authorized representative of the insurer. Section 301 of the Insurance Code which provides as follows: "
ISSUE: Section 301. Any person who for any compensation,
commission or other thing of value, acts or aids in soliciting,
WON the insurance contract was already in effect when the negotiating or procuring the making of any insurance contact or
vessel sank rendering South Sea Surety liable to Hardwood. in placing risk or taking out insurance, on behalf of an insured
other than himself , shall be an insurance broker within the The subject policies are valid and binding if the premium were
intent of this Code, and shall thereby become liable to all the paid on installments. Insurer may also grant credit extension for
duties, requirements, liabilities and penalties to which an payment of the premium.
insurance broker is subject'". The appellees, upon the other
Where the risk is entire and the contract is indivisible, the
hand, claim that the second paragraph of Section 306 of the
insured is not entitled to a refund of the premiums paid if the
Insurance Code provides as follows: 'Section 306. . . . Any
insurer was exposed to the risk insured for any period however
insurance company which delivers to an insurance agent or
brief or momentary.
insurance broker a policy or contract of insurance shall be
deemed to have authorized such agent or broker to receive on
its behalf payment of any premium which is due on such policy
of contract of insurance at the time of its issuance or delivery or FACTS
which becomes due thereon.'" On cross-examination in behalf American Home Assurance Co. (AHAC), represented by
of South Sea Surety and Insurance Co., Inc., Mr. Chua testified American International Underwriters (Phils.), Inc., issued in
that the marine cargo insurance policy for the plaintiff's logs was favor of petitioner Makati Tuscany Condominium Corporation
delivered to him on 21 January 1984 at his office to be delivered an insurance policy on the latter's building and premises, for the
to the plaintiff. When the appellant South Sea Surety and period 1 March 1982 to1 March 1983. The premium was paid
Insurance Co., Inc. delivered to Mr. Chua the marine cargo on installments all of which were accepted by AHAC.
insurance policy for the plaintiff's logs, he is deemed to have
been authorized by the South Sea Surety and Insurance Co., A second policy was issued to renew the first one, this time
Inc. to receive the premium which is due on its behalf. When covering the period 1 March 1983 to 1 March 1984. This was
therefore the insured logs were lost, the insured had already also pain in installment basis. A third policy was again issued
paid the premium to an agent of the South Sea Surety and for the period 1 March 1984 to 1 March 1985. For this, petitioner
Insurance Co., Inc., which is consequently liable to pay the made two installment payments, both accepted by AHAC.
insurance proceeds under the policy it issued to the insured. Thereafter, petitioner refused to pay the balance of the
premium. AHAC filed an action to recover the unpaid balance
of P314,103.05.
15. MAKATI TUSCANY CONDOMINIUM V. CA Petitioner explained that it discontinued the payment of
215 SCRA 462 premiums because the policy did not contain a credit clause in
its favor and the receipts for the installment payments covering
BELLOSILLO; November 6, 1992 the policy for 1984-85, as well as the two (2) previous policies,
stated the following reservations:
2. Acceptance of this payment shall not waive any of the
DOCTRINE:
company rights to deny liability on any claim under the policy
arising before such payments or after the expiration of the credit express acknowledgment in the policies of such receipt of the
clause of the policy; and corresponding premium payments, and petitioner's failure to
pay said premiums on or before the effective dates of said
3. Subject to no loss prior to premium payment If there be any
policies rendered them invalid.
loss such is not covered.
Petitioner thus concludes that there cannot be a perfected
Petitioner further claimed that the policy was never binding and
contract of insurance upon mere partial payment of the
valid, and no risk attached to the policy. It then pleaded a
premiums because under Sec. 77 of the Insurance Code, no
counterclaim for P152k for the premiums already paid for 1984-
contract of insurance is valid and binding unless the premium
85, and in its answer with amended counterclaim, sought the
thereof has been paid, notwithstanding any agreement to the
refund of P924,206.10 representing the premium payments for
contrary.
1982-85.
Trial court dismissed the complaint and the counterclaim upon
the following findings: (1) payment of the premiums of the three ISSUE
policies were made during the term of said policies, hence, it
WON payment by installment of the premiums due on an
could not be said, inspite of the reservations, that no risk
insurance policy invalidates the contract of insurance
attached under the policies; (2) as regards the unpaid
premiums, in view of the reservation in the receipts ordinarily
issued by AHAC on premium payments the only plausible
conclusion is that AHAC has no right to demand their payment HELD
after the lapse of the term of said policy on March 1, 1985. Where the risk is entire and the contract is indivisible, the
Therefore, Tuscany was justified in refusing to pay the same. insured is not entitled to a refund of the premiums paid if the
CA modified the decision by ordering Tuscany to pay the insurer was exposed to the risk insured for any period, however
balance of the premiums due on the third policy plus legal brief or momentary.
interest until fully paid, and affirming the denial of the Reasoning
counterclaim.
The obligation to pay premiums when due is ordinarily as
indivisible obligation to pay the entire premium. Here, the
Petitioner’s Claims parties herein agreed to make the premiums payable in
installments, and there is no pretense that the parties never
Petitioner argues that where the premiums is not actually paid envisioned to make the insurance contract binding between
in full, the policy would only be effective if there is an them. And the insured never informed the insurer that it was
acknowledgment in the policy of the receipt of premium terminating the policy because the terms were unacceptable.
pursuant to Sec. 78 of the Insurance Code. The absence of an
There is nothing in Section 77 which suggests that the parties 356 SCRA 307
may not agree to allow payment of the premiums in installment,
DAVIDE; April 4, 2001
or to consider the contract as valid and binding upon payment
of the first premium.
The records clearly show that petitioner and private respondent DOCTRINE:
intended subject insurance policies to be binding and effective
notwithstanding the staggered payment of the premiums. Exceptions to the rule in Section 77 that there be prepayment
Acceptance of payments speaks loudly of the insurer's intention of premiums as a condition to the validity of the insurance
to honor the policies it issued to petitioner. contract:
in case of life or industrial life policy whenever the grace period
provision applies;
Section 78 of the Insurance Code in effect allows waiver by the
insurer of the condition of prepayment by making an Sec. 78 – any acknowledgement in a policy contract of
acknowledgment in the insurance policy of receipt of premium Insurance of the receipt of the premium is conclusive evidence
as conclusive evidence of payment so far as to make the policy of its payment so far as to make the policy binding,
binding despite the fact that premium is actually unpaid. Section notwithstanding any stipulation therein that it shall not be
77 merely precludes the parties from stipulating that the policy binding until premium is actually paid;
is valid even if premiums are not paid, but does not expressly If the parties agreed to the payment in installments of the
prohibit an agreement granting credit extension, and such an premium and partial payment has been made at the time of the
agreement is not contrary to morals, good customs, public order loss;
or public policy. At the very least, both parties should be
deemed in estoppel to question the arrangement they have The insurer may grant credit extension for the payment of the
voluntarily accepted. premium; and
Estoppel.

Disposition Judgment affirmed. Costs against petitioner.


There is nothing in Sec. 77 which prohibits the parties in an
insurance contract to provide a credit term within which to pay
the premiums.
16. UCPB GENERAL INSURANCE CO., INC. v.
MASAGANA TELAMART, INC. (EN BANC) NATURE
Motion for reconsideration of the decision of the Supreme Court. actual payment. It urges the Court to take judicial notice of the
fact that despite the express provision of Section 77 of the
Insurance Code, extension of credit terms in premium payment
FACTS has been the prevalent practice in the insurance industry. Most
insurance companies, including Petitioner, extend credit terms
In its decision of 15 June 1999, the SC defined the main issue because Section 77 of the Insurance Code is not a prohibitive
to be “whether the fire insurance policies issued by petitioner to
the respondent covering the period from May 22, 1991 to May injunction but is merely designed for the protection of the parties
22, 1992 had been extended or renewed by an implied credit to an insurance contract. The Code itself, in Section 78,
arrangement though actual payment of premium was tendered authorizes the validity of a policy notwithstanding non-payment
on a later date and after the occurrence of the (fire) risk insured of premiums.
against.” The Court resolved this issue in the negative in view
Respondent also asserts that the principle of estoppel applies
of Section 77 of the Insurance Code and its decisions in
to Petitioner. Despite its awareness of Section 77 Petitioner
Valenzuela v. Court of Appeals; South Sea Surety and
persuaded and induced Respondent to believe that payment of
Insurance
premium on the 60- to 90-day credit term was perfectly alright;
Co., Inc. v. Court of Appeals; and Tibay v. Court of Appeals. in fact it accepted payments within 60 to 90 days after the due
Accordingly, it reversed and set aside the decision of the Court dates. By extending credit and habitually accepting payments
of Appeals. 60 to 90 days from the effective dates of the policies, it has
implicitly agreed to modify the tenor of the insurance policy and
Respondent seasonably filed a motion for the reconsideration in effect waived the provision therein that it would pay only for
of the adverse verdict. It alleges in the motion that the SC had the loss or damage in case the same occurred after payment of
made in the decision its own findings of facts, which are not in the premium.
accord with those of the trial court and the Court of Appeals.
The courts below correctly found that no notice of non-renewal Petitioner filed an opposition to the Respondent’s motion for
was made within 45 days before 22 May 1992, or before the reconsideration. It argues that both the trial court and the Court
expiration date of the fire insurance policies. Thus, the policies of Appeals overlooked the fact that on 6 April 1992 Petitioner
in question sent by ordinary mail to Respondent a notice of non-renewal
and sent by personal delivery a copy thereof to Respondent’s
were renewed by operation of law and were effective and valid broker, Zuellig. Both courts likewise ignored the fact that
on 30 June 1992 when the fire occurred, since the premiums Respondent was fully aware of the notice of non-renewal. A
were paid within the 60- to 90- day credit term. reading of Section 66 of the Insurance Code readily shows that
Respondent likewise disagrees with its ruling that parties may in order for an insured to be entitled to a renewal of a non-life
neither agree expressly or impliedly on the extension of credit policy, payment of the premium due on the effective date of
or time to pay the premium nor consider a policy binding before renewal should first be made. Respondent’s argument that
Section 77 is not a prohibitive provision finds no authoritative SEC. 77. An insurer is entitled to payment of the premium as
support. soon as the thing insured is exposed to the peril insured against.
Notwithstanding any agreement to the contrary, no policy or
The following facts, as found by the trial court and the Court of
contract of insurance issued by an insurance company is valid
Appeals, are indeed duly established:
and binding unless and until the premium thereof has been paid,
1. For years, Petitioner had been issuing fire policies to the except in the case of a life or an industrial life policy whenever
Respondent, and these policies were annually renewed. the grace period provision applies.

2. Petitioner had been granting Respondent a 60- to 90-day This Section is a reproduction of Section 77 of P.D. No. 612
credit term within which to pay the premiums on the renewed (The Insurance Code) promulgated on 18 December 1974. In
policies. turn, this Section has its source in Section 72 of Act No. 2427
otherwise known as the Insurance Act as amended by R.A. No.
3. There was no valid notice of non-renewal of the policies in 3540, approved on 21 June 1963, which read:
question, as there is no proof at all that the notice sent by
ordinary mail was received by Respondent, and the copy SEC. 72. An insurer is entitled to payment of premium as soon
thereof allegedly sent to Zuellig was ever transmitted to as the thing insured is exposed to the peril insured against,
Respondent. unless there is clear agreement to grant the insured credit
extension of the premium due. No policy issued by an insurance
4. The premiums for the policies in question in the aggregate company is valid and binding unless and until the premium
amount of P225,753.95 were paid by Respondent within the 60- thereof has been paid.
to 90-day credit term and were duly accepted and received by
Petitioner’s cashier. - It can be seen at once that Section 77 does not restate the
portion of Section 72 expressly permitting an agreement to
extend the period to pay the premium. But there are exceptions
ISSUE: to Section 77.

WON Sec. 77 of the Insurance Code of 1978 must be strictly The first exception is provided by Section 77 itself, and that is,
applied to Petitioner’s advantage despite its in case of a life or industrial life policy whenever the grace period
provision applies. The second is that covered by Section 78 of
practice of granting a 60- to 90-day credit term for the payment the Insurance Code, which provides:
of premiums
SEC. 78. Any acknowledgment in a policy or contract of
HELD insurance of the receipt of premium is conclusive evidence of
its payment, so far as to make the policy binding,
NO
notwithstanding any stipulation therein that it shall not be
- Section 77 of the Insurance Code of 1978 provides: binding until premium is actually paid.
- A third exception was laid down in Makati Tuscany Disposition Judgment reconsidered and set aside, that of the
Condominium Corporation vs. Court of Appeals, wherein we Court of Appeals affirmed in toto.
ruled that Section 77 may not apply if the parties have agreed
to the payment in installments of the premium and partial
payment has been made at the time of loss. Tuscany has SEPARATE OPINION
provided a fourth exception to Section 77, namely, that the
insurer may grant credit extension for the payment of the VITUG
premium. This simply means that if the insurer has granted the - An essential characteristic of an insurance is its being
insured a credit term for the payment of the premium and loss synallagmatic, a highly reciprocal contract where the rights and
occurs before the expiration of the term, recovery on the policy obligations of the parties correlate and mutually correspond.
should be allowed even though the premium is paid after the
loss but within the credit term. Moreover, there is nothing in - By weight of authority, estoppel cannot create a contract of
Section 77 which prohibits the parties in an insurance contract insurance, neither can it be successfully invoked to create a
to provide a credit term within which to pay the premiums. That primary liability, nor can it give validity to what the law so
agreement is not against the law, morals, good customs, public procribes as a matter of public policy.
order or public policy. The agreement binds the parties. Article
PARDO [dissent]
1306 of the Civil Code provides:
- An assured’s failure to give notice of the fire immediately upon
ART. 1306. The contracting parties may establish such
its occurrence blatantly showed the fraudulent character of its
stipulations clauses, terms and conditions as they may deem
claims. Respondent is required by law and by express terms of
convenient, provided they are not contrary to law, morals, good
the policy to give immediate written notice of loss. This must be
customs, public order, or public policy.
complied with in the utmost good faith.
- Assuming arguendo that the 60- to 90-day credit has been
Finally, it would be unjust and inequitable if recovery on the agreed between the parties, respondent could not still invoke
policy would not be permitted against Petitioner, which had estoppel to back up its claim. Estoppel cannot give validity to an
consistently granted a 60- to 90-day credit term for the payment act that is prohibited by law or against public policy. The actual
of premiums despite its full awareness of Section 77. Estoppel payment of premiums is a condition precedent to the validity of
bars it from taking refuge under said Section since Respondent an insurance contract other than life insurance policy. Any
relied in good faith on such agreement to the contrary is void as against law and public
policy.
practice. Estoppel then is the fifth exception to Section 77.
17. AMERICAN HOME ASSURANCE Company of the Philippines. Petitioner refused to honor the
COMPANY, petitioner, vs. ANTONIO CHUA, respondent. claim notwithstanding several demands by respondent, thus,
the latter filed an action against petitioner before the trial court.
G.R. No. 130421. June 28, 1999
In its defense, petitioner claimed there was no existing
DAVIDE, JR., C .J|||
insurance contract when the fire occurred since respondent did
DOCTRINE: not pay the premium. It also alleged that even assuming there
was a contract, respondent violated several conditions of the
policy, particularly: (1) his submission of fraudulent income tax
FACTS: return and financial statements; (2) his failure to establish the
actual loss, which petitioner assessed at P70,000; and (3) his
Petitioner is a domestic corporation engaged in the insurance failure to notify to petitioner of any insurance already effected to
business. Sometime in 1990, respondent obtained from cover the insured goods. These violations, petitioner insisted,
petitioner a fire insurance covering the stock-in-trade of his justified the denial of the claim.
business, Moonlight Enterprises, located at Valencia, Bukidnon.
The insurance was due to expire on 25 March 1990. The trial court ruled in favor of respondent. It found that
respondent paid by way of check a day before the fire occurred.
On 5 April 1990 respondent issued PCIBank Check No. 352123 As to respondent's failure to notify petitioner of the other
in the amount of P2,983.50 to petitioner's agent, James Uy, as insurance contracts covering the same goods, the trial court
payment for the renewal of the policy. In turn, the latter delivered held that petitioner failed to show that such omission was
Renewal Certificate No. 00099047 to respondent. The check intentional and fraudulent. On appeal, the assailed decision was
was drawn against a Manila bank and deposited in petitioner's affirmed in toto by the Court of Appeals. Its motion for
bank account in Cagayan de Oro City. The corresponding reconsideration of the judgment having been denied, petitioner
official receipt was issued on 10 April. Subsequently, a new filed the petition in this case. Petitioner reiterates its stand that
insurance policy, Policy No. 206-4234498-7, was issued, there was no existing insurance contract between the parties. It
whereby petitioner undertook to indemnify respondent for any invokes Section 77 of the Insurance Code|||. Petitioner
damage or loss arising from fire up to P200,000 for the period emphasizes that when the fire occurred on 6 April 1990 the
25 March 1990 to 25 March 1991. insurance contract was not yet subsisting pursuant toArticle
1249 3 of the Civil Code, which recognizes that a check can
On 6 April 1990 Moonlight Enterprises was completely razed by
only effect payment once it has been cashed. Although
fire. Total loss was estimated between P4,000,000 and
respondent testified that he gave the check on 5 April to a
P5,000,000. Respondent filed an insurance claim with petitioner
certain James Uy, the check, drawn against a Manila bank and
and four other co-insurers, namely, Pioneer Insurance and
deposited in a Cagayan de Oro City bank, could not have been
Surety Corporation, Prudential Guarantee and Assurance, Inc.,
cleared by 6 April, the date of the fire. In fact, the official receipt
Filipino Merchants Insurance Co. and Domestic Insurance
issued for respondent's check payment was dated 10 April the instant case, the best evidence of such authority is the fact
1990, four days after the fire occurred. that petitioner accepted the check and issued the official receipt
for the payment. It is, as well, bound by its agent's
petitioner also contends that respondent's non-disclosure of the
acknowledgment of receipt of payment.||| Section 78 of the
other insurance contracts rendered the policy void.
Insurance Code explicitly provides:
ISSUE:
An acknowledgment in a policy or contract of insurance of the
WON there was a valid payment of premium, considering that receipt of premium is conclusive evidence of its payment, so far
respondent's check was cashed after the occurrence of the as to make the policy binding, notwithstanding any stipulation
fire|||. therein that it shall not be binding until the premium is actually
paid.
WON the non-disclosure of other insurance contracts render the
policy void. This Section establishes a legal fiction of payment and should
be interpreted as an exception to Section 77
HELD:
Ordinarily, where the insurance policy specifies as a condition
YES. the disclosure of existing co-insurers, non-disclosure thereof is
The general rule in insurance laws is that unless the premium a violation that entitles the insurer to avoid the policy. This
is paid the insurance policy is not valid and binding. The only condition is common in fire insurance policies and is known as
exceptions are life and industrial life insurance. 6 Whether the "other insurance clause." The purpose for the inclusion of
payment was indeed made is a question of fact which is best this clause is to prevent an increase in the moral hazard. We
determined by the trial court. The trial court found, as affirmed have ruled on its validity and the case of Geagonia v. Court of
by the Court of Appeals, that there was a valid check payment Appeals 10 clearly illustrates such principle. However, we see
by respondent to petitioner. the renewal certificate issued to an exception in the instant case.
respondent contained the acknowledgment that premium had Citing Section 29 11 of the Insurance Code, the trial court
been paid. It is not disputed that the check drawn by respondent reasoned that respondent's failure to disclose was not
in favor of petitioner and delivered to its agent was honored intentional and fraudulent. The application of Section 29 is
when presented and petitioner forthwith issued its official receipt misplaced. Section 29 concerns concealment which is
to respondent on 10 April 1990. Section 306 of the Insurance intentional. The relevant provision is Section 75, which provides
Code provides that any insurance company which delivers a that:
policy or contract of insurance to an insurance agent or
insurance broker shall be deemed to have authorized such A policy may declare that a violation of specified provisions
agent or broker to receive on its behalf payment of any premium thereof shall avoid it, otherwise the breach of an immaterial
which is due on such policy or contract of insurance at the time provision does not avoid the policy.
of its issuance or delivery or which becomes due thereon. 8 In
To constitute a violation the other existing insurance contracts
must be upon the same subject matter and with the same
FACTS:
interest and risk. 12 Indeed, respondent acquired several co-
insurers and he failed to disclose this information to petitioner. Since 1989, Wyeth Philippines, Inc. (Wyeth) and respondent
Nonetheless, petitioner is estopped from invoking this Reputable Forwarder Services, Inc. (Reputable) had been
argument. The trial court cited the testimony of petitioner's loss annually executing a contract of carriage, whereby the latter
adjuster who admitted previous knowledge of the co-insurers. undertook to transport and deliver the former’s products to its
customers, dealers or salesmen. On November 18, 1993,
Wyeth procured Marine Policy No. MAR 13797 (Marine Policy)
from respondent Philippines First Insurance Co., Inc.
(Philippines First) to secure its interest over its own products.
Philippines First thereby insured Wyeth’s nutritional,
pharmaceutical and other products usual or incidental to the
insured’s business while the same were being transported or
shipped in the Philippines. The policy covers all risks of direct
18. MALAYAN INSURANCE CO., INC., petitioner, vs. physical loss or damage from any external cause, if by land, and
PHILIPPINES FIRST INSURANCE CO., INC. and provides a limit of P6,000,000.00 per any one land vehicle.
REPUTABLE FORWARDER SERVICES, INC., respondents. On December 1, 1993, Wyeth executed its annual contract of
G.R. No. 184300. July 11, 2012||| carriage with Reputable. It turned out, however, that the
contract was not signed by Wyeth’s representative/s.
REYES, J. Nevertheless, it was admittedly signed by Reputable’s
representatives, the terms thereof faithfully observed by the
parties and, as previously stated, the same contract of carriage
DOCTRINE: had been annually executed by the parties every year since
1989. Under the contract, Reputable undertook to answer for
Double insurance exists where the same person is insured by “all risks with respect to the goods and shall be liable to the
several insurers separately in respect to the same subject and COMPANY (Wyeth), for the loss, destruction, or damage of the
interest. goods/products due to any and all causes whatsoever, including
. A contract of insurance, being a contract of adhesion, par theft, robbery, flood, storm, earthquakes, lightning, and other
excellence, any ambiguity therein should be resolved against force majeure while the goods/products are in transit and until
the insurer. actual delivery to the customers, salesmen, and dealers of the
COMPANY”. The contract also required Reputable to secure an
insurance policy on Wyeth’s goods.
Thus, on February 11, 1994, Reputable signed a Special Risk 5. There is identity of the risk or peril insured against.
Insurance Policy (SR Policy) with petitioner Malayan for the
amount of P1,000,000.00. On October 6, 1994, during the
effectivity of the Marine Policy and SR Policy, Reputable In the present case, while it is true that the Marine Policy and
received from Wyeth 1,000 boxes of Promil infant formula worth the SR Policy were both issued over the same subject matter,
P2,357,582.70 to be delivered by Reputable to Mercury Drug i.e. goods belonging to Wyeth, and both covered the same peril
Corporation in Libis, Quezon City. Unfortunately, on the same insured against, it is, however, beyond cavil that the said
date, the truck carrying Wyeth’s products was hijacked by about policies were issued to two different persons or entities. It is
10 armed men. They threatened to kill the truck driver and two undisputed that Wyeth is the recognized insured of Philippines
of his helpers should they refuse to turn over the truck and its First under its Marine Policy, while Reputable is the recognized
contents to the said highway robbers. The hijacked truck was insured of Malayan under the SR Policy. The fact that
recovered two weeks later without its cargo. Malayan questions Reputable procured Malayan’s SR Policy over the goods of
its liability based on sections 5 and 12 of the SR Policy. Wyeth pursuant merely to the stipulated requirement under its
contract of carriage with the latter does not make Reputable a
ISSUE: mere agent of Wyeth in obtaining the said SR Policy.
WON there is double insurance in this case such that either
Section 5 or Section 12 of the SR Policy may be applied. The interest of Wyeth over the property subject matter of both
WON there was over insurance. insurance contracts is also different and distinct from that of
Reputable’s. The policy issued by Philippines First was in
HELD: consideration of the legal and/or equitable interest of Wyeth
over its own goods. On the other hand, what was issued by
1. No.
Malayan to Reputable was over the latter’s insurable interest
By the express provision of Section 93 of the Insurance Code, over the safety of the goods, which may become the basis of
double insurance exists where the same person is insured by the latter’s liability in case of loss or damage to the property and
several insurers separately in respect to the same subject and falls within the contemplation of Section 15 of the Insurance
interest. The requisites in order for double insurance to arise are Code.
as follows: Therefore, even though the two concerned insurance policies
1. The person insured is the same; were issued over the same goods and cover the same risk,
2. Two or more insurers insuring separately; there arises no double insurance since they were issued to two
3. There is identity of subject matter; different persons/entities having distinct insurable interests.
Necessarily, over insurance by double insurance cannot
4. There is identity of interest insured; and
likewise exist. Hence, as correctly ruled by the RTC and CA,
neither Section 5 nor Section 12 of the SR Policy can be merely acknowledged, on behalf of the insurance company,
applied. that the latter's branch office had received from the applicant
the insurance premium and had accepted the application
2. NO. subject for processing by the insurance company, such binding
deposit receipt does not become in force until the application is
Sec. 96 presupposes over insurance due to double insurance. approved.
Since there was no double insurance here, over insurance
cannot then exist. FACTS:
On March 14, 1957, private respondent Ngo Hing filed an
application with Great Pacific Life Assurance Company for a
Indemnity and liability insurance policies are construed in twenty-year endowment policy in the amount of P50, 000.00 on
accordance with the general rule of resolving any ambiguity the life of his one-year old daughter Helen Go. Ngo Hing
therein in favor of the insured, where the contract or policy is supplied the essential data which petitioner Lapu-lapu D.
prepared by the insurer. A contract of insurance, being a Mondragon, Branch Manager of the Pacific Life in Cebu City
contract of adhesion, par excellence, any ambiguity therein wrote on the corresponding form in his own handwriting.
should be resolved against the insurer; in other words, it should Petitioner Mondragon handwrote at the bottom of the back
page of the application form his strong recommendation for the
be construed liberally in favor of the insured and strictly against
approval of the insurance application. Then on April 30, 1957,
the insurer. Limitations of liability should be regarded with Mondragon received a letter from Pacific Life disapproving the
extreme jealousy and must be construed in such a way as to insurance application. The letter stated that the
preclude the insurer from noncompliance with its said life insurance application for 20-year endowment plan is
obligations.||| To rule that Sec. 12 operates even in the absence not available for minors below seven years old, but
of double insurance would work injustice to Reputable which, Pacific Life can consider the same under the Juvenile Triple
despite paying premiums for a [P]1,000,000.00 insurance Action Plan, and advised that if the offer is acceptable, the
coverage, would not be entitled to recover said amount for the Juvenile Non-Medical Declaration be sent to the Company.
simple reason that the same property is covered by another The non-acceptance of the insurance plan by Pacific Life was
insurance policy, a policy to which it was not a party to and much allegedly not communicated by petitioner Mondragon to private
less, from which it did not stand to benefit.||| respondent Ngo Hing. Instead, on May 6, 1957, Mondragon
wrote back Pacific Life again strongly recommending the
approval of the 20-year endowment life insurance on the
19. Great Pacific Life Insurance v. CA ground that Pacific Life is the only insurance company not
selling the 20-year endowment insurance plan to children,
DOCTRINE: pointing out that since 1954 the customers, especially the
Chinese, were asking for such coverage.
Where the binding deposit receipt is intended to be merely a
provisional or temporary insurance contract, and that the receipt
It was when things were in such state that on May 28, 1957 DOCTRINE:
Helen Go died of influenza with complication of broncho-
When the requirements for a ridder is complied with, it is
pneumonia. Thereupon, private respondent sought the
considered part of the policy.
payment of the proceeds of the insurance, but having failed in
his effort, he filed the action for the recovery of the same before FACTS:
the Court of First Instance of Cebu.
Private respondent Lincoln Philippine Life Insurance Co., Inc.,
ISSUE: (now Jardine-CMA Life Insurance Company, Inc.) is a domestic
corporation registered with the Securities and Exchange
Whether or not a binding receipt constitute temporary contract Commission and engaged in life insurance business. In the
of insurance years prior to 1984, private respondent issued a special kind of
HELD: life insurance policy known as the "Junior Estate Builder Policy,"
the distinguishing feature of which is a clause providing for an
NO. Where the binding deposit receipt is intended to be merely automatic increase in the amount of life insurance coverage
a provisional or temporary insurance contract, and that the upon attainment of a certain age by the insured without the need
receipt merely acknowledged, on behalf of the insurance of issuing a new policy. The clause was to take effect in the year
company, that the latter's branch office had received from the 1984. Documentary stamp taxes due on the policy were paid by
applicant the insurance premium and had accepted the petitioner only on the initial sum assured. In 1984, private
application subject for processing by the insurance company, respondent also issued 50,000 shares of stock dividends with a
such binding deposit receipt does not become in force until the par value of P100.00 per share. Subsequently, petitioner, CIR,
application is approved. A binding deposit receipt which is issued deficiency documentary stamps tax assessment for the
merely conditional does not insure outright. Thus, where an year 1984, in the amounts of (a) P464,898.75, corresponding to
agreement is made between the applicant and the agent, no the amount of automatic increase of the sum assured on the
liability will attack until the principal approves the risk and a policy issued by respondent, and (b) P78,991.25 corresponding
receipt is given by the agent. The acceptance is merely to the book value in excess of the par value of the stock
conditional, and is subordinated to the act of the company in dividends.
approving or rejecting the application. A contract of insurance,
Private respondent questioned the deficiency assessments and
like other contracts, must be assented to by both parties either
sought their cancellation. On March 30, 1993, the Court of Tax
in person or by their agents. The contract, to be binding from
Appeals found no valid basis for the deficiency tax assessment
the date of the application, must have been a completed
on the stock dividends, as well as on the insurance policy.
contract, one that leaves nothing to be done, nothing to be
Petitioner appealed the CTA's decision to the Court of Appeals.
completed, nothing to be passed upon, or determined, before it
On November 18, 1994, the Court of Appeals promulgated a
shall take effect.
decision affirming the CTA's decision insofar as it nullified the
deficiency assessment on the insurance policy, but reversing
the same with regard to the deficiency assessment on the stock
20. CIR v. Lincoln Philippine Life Insurance Company dividends.
ISSUE: been held "that the concealment must, in the absence of
inquiries, be not only material, but fraudulent, or the fact must
Whether or not "automatic increase clause" in the subject
have been intentionally withheld.
insurance policy is separate and distinct from the main
agreement and involves another transaction. FACTS:
HELD: On May 12, 1962, Kwong Nam applied for a 20-year
endowment insurance on his life for the sum of P20,000.00,
NO. Section 49, Title VI of the Insurance Code defines an with his wife, appellee Ng Gan Zee, as beneficiary. On the
insurance policy as the written instrument in which a contract of same date, appellant, upon receipt of the required premium
insurance is set forth. Section 50 of the same Code provides from the insured, approved the application and issued the
that the policy, which is required to be in printed form, may corresponding policy. On December 6, 1963, Kwong Nam died
contain any word, phrase, clause, mark, sign, symbol, of cancer of the liver with metastasis. All premiums had been
signature, number, or word necessary to complete the contract religiously paid at the time of his death. On January 10, 1964,
of insurance. It is thus clear that any rider, clause, warranty or his widow Ng Gan Zee presented a claim in due form to
endorsement pasted or attached to the policy is considered part appellant for payment of the face value of the policy. On the
of such policy or contract of insurance. same date, she submitted the required proof of death of the
A rider is an attachment to an insurance policy that modifies the insured. Appellant denied the claim on the ground that the
condition of the policy by expanding or restricting its benefits or answers given by the insured to the questions appearing in his
excluding certain conditions from the coverage. Riders, together application for life insurance were untrue
with other attachments to the policy such as clauses, warranties ISSUE:
or endorsements are not binding on the insured unless the
descriptive title or name thereof is mentioned and written on the Whether or not appellant, because of insured's aforesaid
blank spaces provided in the policy. When the requirements for representation, into misled or deceived entering the contract or
a ridder are complied with, it is considered part of the policy. It in accepting the risk at the rate of premium agreed upon
is not a separate contract.
HELD:
NO. Section 27 of the Insurance Law [Act 2427] provides:
21. Ng Gan Zee v. Asian Crusader Life Assurance
"Sec. 27. Such party to a contract of insurance must
Corporation
communicate to the other, in good faith, all facts within
DOCTRINE: his knowledge which are material to the contract, and
which the other has not the means of ascertaining, and
Concealment exists where the assured had knowledge of a fact as to which he makes no warranty."
material to the risk, and honesty, good faith, and fair dealing
requires that he should communicate it to the assurer, but he Thus, "concealment exists where the assured had knowledge
designedly and intentionally withholds me same." It has also of a fact material to the risk, and honesty, good faith, and fair
dealing requires that he should communicate it to the assurer, DOCTRINE:
but he designedly and intentionally withholds the same." “that
The burden that the loss was caused by an excepted peril lies
the concealment must, in the absence of inquiries, be not only
in the insurer. If such terms are clear and unambiguous, they
material, but fraudulent, or the fact must have been intentionally
must be taken and understood in their plain, ordinary and
withheld."
popular sense.
Assuming that the aforesaid answer given by the insured is
FACTS:
false, as claimed by the appellant. Sec. 27 of the Insurance Law,
above-quoted, nevertheless requires that fraudulent intent on On February 21, 2007, respondent entered into a contract
the part of the insured be established to entitle the insurer to of insurance, Motor Car Policy, with petitioner, involving her
rescind the contract. And as correctly observed by the lower motor vehicle, a Toyota Revo DLX DSL. The contract
court, "misrepresentation as a defense of the insurer to avoid of insurance obligates the petitioner to pay the respondent the
liability is an 'affirmative' defense. The duty to establish such a amount of Six Hundred Thirty Thousand Pesos (P630,000.00)
defense by satisfactory and convincing evidence rests upon the in case of loss or damage to said vehicle during the period
defendant. The evidence before the Court does not clearly and covered, which is from February 26, 2007 to February 26, 2008.
satisfactorily establish that defense." On April 16, 2007, at about 9:00 a.m., respondent instructed her
driver, Jose Joel Salazar Lanuza (Lanuza), to bring the above-
Kwong Nam had informed the appellant's medical examiner that
described vehicle to a nearby auto-shop for a tune-up.
the tumor for which he was operated on was ''associated with
However, Lanuza no longer returned the motor vehicle to
ulcer of the stomach." In the absence of evidence that the
respondent and despite diligent efforts to locate the same, said
insured had sufficient medical knowledge as to enable him to
efforts proved futile. Resultantly, respondent promptly reported
distinguish between "peptic ulcer" and "a tumor", his statement
the incident to the police and concomitantly notified petitioner of
that said tumor was "associated with ulcer of the stomach,”
the said loss and demanded payment of
should be construed as an expression made in good faith of his
the insurance proceeds in the total sum of P630,000.00. In a
belief as to the nature of his ailment and operation. Indeed, such
letter dated July 5, 2007, petitioner denied the insurance claim
statement must be presumed to have been made by him without
of respondent. In letters dated July 12, 2007 and August 3,
knowledge of its incorrectness and without any deliberate intent
2007, respondent reiterated her claim and argued that the
on his part to mislead the appellant.
exception refers to damage of the motor vehicle and not to its
While it may be conceded that, from the viewpoint of a medical loss. However, petitioner's denial of respondent's insured claim
expert, the information communicated was imperfect, the same remains firm.
was nevertheless sufficient to have induced appellant to make ISSUE:
further inquiries about the ailment and operation of the insured.
Whether or not the loss of respondent's vehicle is excluded
under the insurance policy
22. Alpha Insurance and Surety co. v. Castor
HELD: FACTS:
NO. Theft perpetrated by the driver of the insured is not an On April 15, 1986, Robert John B. Bacani procured a life
exception to the coverage from the insurance policy. Contracts insurance contract for himself from petitioner. He was issued
of insurance, like other contracts, are to be construed according Policy valued P100,000.00, with double indemnity in
to the sense and meaning of the terms which the parties case of accidental death. The designated beneficiary was his
themselves have used. If such terms are clear and mother, respondent Bernarda Bacani. On June 26, 1987, the
unambiguous, they must be taken and understood in their plain, insured died in a plane crash. Respondent Bernarda Bacani
filed a claim with petitioner, seeking the benefits of the
ordinary and popular sense. Accordingly, in interpreting the
insurance policy taken by her son. Petitioner conducted an
exclusions in an insurance contract, the terms used specifying
investigation and its findings prompted it to reject the claim. In
the excluded classes therein are to be given their meaning as its letter, petitioner informed respondent Bernarda Bacani,
understood in common speech. Adverse to petitioner's claim, that the insured did not disclosed material facts relevant to the
the words "loss" and "damage" mean different things in common issuance of the policy, thus rendering the
ordinary usage. The word "loss" refers to the act or fact of contract of insurance voidable.
losing, or failure to keep possession, while the word "damage"
Petitioner claimed that the insured gave false statements in
means deterioration or injury to property. When the terms of the his application, Petitioner discovered that two weeks prior to
insurance policy are ambiguous, equivocal or uncertain, such his application for insurance, the insured was examined and
that the parties themselves disagree about the meaning of confined at the Lung Center of the Philippines, where he was
particular provisions, the policy will be construed by the courts diagnosed for renal failure. During his confinement, the
liberally in favor of the assured and strictly against the insurer. deceased was subjected to urinalysis, ultra-sonography and
A contract of insurance is a contract of adhesion. So, when the hematology tests.
terms of the insurance contract contain limitations on liability, In the ruling of private respondents, the trial court concluded
courts should construe them in such a way as to preclude the that the facts concealed by the insured were made in good
insurer from non-compliance with his obligation. faith and under the belief that they need not be disclosed.
Moreover, it held that the health history of the insured was
immaterial since the insurance policy was "non-medical." The
23. Sunlife Assurance Company of Canada v. CA appellate court ruled that petitioner cannot avoid its obligation
by claiming concealment because the cause of death was
DOCTRINE: unrelated to the facts concealed by the insured.
Materiality is to be determined not by the event, but solely by ISSUE:
the probable and reasonable influence of the facts upon the Whether or not the contract of insurance is enforceable
party to whom communication is due, in forming his despite the false statement given in his application
estimate of the disadvantages of the proposed contract or in
making his inquiries. HELD:
NO. Section 26 of the Insurance Code is explicit in requiring a insurer in forming his estimates of the risks of the proposed
party to a contract of insurance to communicate to the other, insurance policy or in making inquiries.
in good faith, all facts within his knowledge which are material
to the contract and as to which he makes no warranty, and FACTS:
which the other has no means of ascertaining. Said Section
Celestino Henson was insured by Philamlife in 1954 upon his
provides:
application or a 20-yr endowment life policy. In 1955, the policy
"A neglect to communicate that which a party lapsed due to non-payment of the premiums. Upon payment of
knows and ought to communicate, is called the premiums due, the policy was reinstated, but in the
concealment."
application for reinstatement, Henson did not disclose the fact
Materiality is to be determined not by the event, but solely by that he had been previously diagnosed for pyelonephritis,
the probable and reasonable influence of the facts upon the enlarged liver and hernia. He also did not disclose that he had
party to whom communication is due, in forming his been examined by a physician. In 1956, Henson died, and his
estimate of the disadvantages of the proposed contract or in beneficiaries’ claim was rejected by Philamlife on the ground of
making his inquiries. concealment.The company then filed for rescission.
Beneficiaries’ contend that the intent to conceal must be proven
The terms of the contract are clear. The insured is specifically to warrant rescission.
required to disclose to the insurer matters relating to his
health. The information which the insured failed to disclose ISSUE:
were material and relevant to the approval and the
issuance of the insurance policy. The matters concealed Whether or not there is need to prove intent to conceal to
would have definitely affected petitioner's action on his warrant rescission.
application, either by approving it with the corresponding
adjustment for a higher premium or rejecting the same. HELD:
Moreover, a disclosure may have warranted a medical NO. Sec. 26 provides that “a concealment whether intentional
examination of the insured by petitioner in order for it to or unintentional entitles the injured party to rescind the contract
reasonably assess the risk involved in accepting the
of insurance”. And aside from this, intent, being a state of the
application.
mind is hard to prove. According to Sec. 30 of the Insurance
Code: Materiality is to be determined not by the event, but
24. Henson v. Philippine American Life Insurance Co., solely by the probable and reasonable influence of the facts
upon the party to whom the communication is due, in forming
DOCTRINE: his estimate of the disadvantages of the proposed contract, or
in making his inquiries. In essence therefore, the insured need
The insured need not die of the disease he had failed to disclose
not have died of the very diseases he had failed to reveal to the
to the insurer. It is sufficient that his non-disclosure misled the
insurance company. It is sufficient that his non-revelation had
misled the insurer in forming its estimate of the disadvantages Canilang to disclose previous medical consultation and
of the proposed policy reinstatement or in making its inquiries, treatment constituted material information which should have
in order to entitle the latter to rescind the contract. been communicated to Great Pacific to enable the latter to
make proper inquiries. Petitioner Thelma Canilang contended
that the non-disclosure of certain facts about his previous health
25. VDA. DE CANILANG vs. CA conditions does not amount to fraud and private respondent is
G.R. No. 92492 June 17, 1993 deemed to have waived inquiry thereto.

DOCTRINE: Sinus tachycardia is considered present when the heart


rate exceeds 100 beats per minute. It is, among others, a
Good faith is not a defense in concealment. A man's common reaction to heart disease, including myocardial
state of mind or subjective belief is not capable of proof in our infarction, and heart failure per se.
judicial process, except through proof of external acts or failure
to act from which inferences as to his subjective belief may be ISSUE:
reasonably drawn.
Whether Jaime Canilang made a material concealment
FACTS: as to the state of his health at the time of the filing of insurance
application
Jaime Canilang consulted Dr. Claudio and was
diagnosed as suffering from sinus tachycardia. Jaime consulted RULING:
again on another day and was found to have acute bronchitis.
Yes. The information which Jaime Canilang failed to
He then applied for a "non-medical" insurance policy with
disclose was material to the ability of Great Pacific to estimate
respondent Great Pacific Life Assurance Company (Great
the probable risk he presented as a subject of life insurance.
Pacific) naming his wife, petitioner Thelma Canilang, as his
Had Canilang disclosed his visits to his doctor, the diagnosis
beneficiary. Jaime was issued ordinary life insurance.
made and the medicines prescribed by such doctor in his
Jaime died of "congestive heart failure," "anemia," and insurance application, it may be reasonably assumed that Great
"chronic anemia." Petitioner, widow and beneficiary of the Pacific would have made further inquiries and would have
insured, filed a claim with Great Pacific which the insurer denied probably refused to issue a non-medical insurance policy or, at
upon the ground that the insured had concealed material the very least, required a higher premium for the same
information from it. Petitioner then filed a complaint against coverage. A man's state of mind or subjective belief is not
Great Pacific with the Insurance Commission for recovery of the capable of proof in our judicial process, except through proof of
insurance proceeds. external acts or failure to act from which inferences as to his
subjective belief may be reasonably drawn.
Insurance Commissioner ordered Great Pacific to pay.
On appeal, the CA reversed and found that the failure of Jaime
Neither does materiality depend upon the actual or accepted, would obviously erase Section 27 from the Insurance
physical events which ensue. Materiality relates rather to the Code of 1978.
"probable and reasonable influence of the facts" upon the party
to whom the communication should have been made, in
assessing the risk involved in making or omitting to make further 26. TAN vs. CA
inquiries and in accepting the application for insurance; that G.R. No. L-48049 June 29, 1989
"probable and reasonable influence of the facts" concealed
must, of course, be determined objectively, by the judge DOCTRINE:
ultimately.
The Incontestability Clause precludes the insurer from
The insurance Great Pacific applied for was a "non- raising the defenses of false representations or concealment of
medical" insurance policy. In Saturnino v. Philippine-American material facts insofar as health and previous diseases are
Life Insurance the Court held that: if anything, the waiver of concerned if the insurance has been in force for at least two
medical examination [in a non-medical insurance contract] years during the insured's lifetime. The phrase "during the
renders even more material the information required of the lifetime" found in Section 48 simply means that the policy is no
applicant concerning previous condition of health and diseases longer considered in force after the insured has died. The key
suffered, for such information necessarily constitutes an phrase in the second paragraph of Section 48 is "for a period of
important factor which the insurer takes into consideration in two years."
deciding whether to issue the policy or not.
FACTS:
Section 27 of the Insurance Code of 1978 is properly
read as referring to "any concealment" without regard to Tan Lee Siong, father of herein petitioners, applied for
whether such concealment is intentional or unintentional. In any life insurance with private respondent The Philippine American
case, in the case at bar, the nature of the facts not conveyed to Life Insurance Company. Said application was approved and
was issued effective November 6, 1973, with petitioners the
the insurer was such that the failure to communicate must have
beneficiaries thereof. On April 26, 1975, Tan Lee Siong died of
been intentional rather than merely inadvertent. For Jaime hepatoma. Petitioners then filed with respondent company their
Canilang could not have been unaware and that he had claim for the proceeds of the life insurance policy. However, in
consulted a doctor twice in the two (2) months before applying a letter dated September 11, 1975, respondent denied the claim
for non-medical insurance. We find it difficult to take seriously and rescinded the policy by reason of the alleged
the argument that Great Pacific had waived inquiry into the misrepresentation and concealment of material facts made by
concealment by issuing the insurance policy notwithstanding the deceased Tan Lee Siong in his application for insurance.
Canilang's failure to set out answers to some of the questions The premiums paid on the policy were thereupon refunded.
in the insurance application. Such failure precisely constituted Alleging that respondent company's refusal to pay them the
concealment on the part of Canilang. Petitioner's argument, if proceeds of the policy was unjustified and unreasonable,
petitioners filed on November 27, 1975, a complaint against the
former with the Office of the Insurance Commissioner. The
Insurance Commissioner and CA dismissed. The insurer has two years from the date of issuance of
the insurance contract or of its last reinstatement within which
The petitioners contended that the respondent company to contest the policy, whether or not, the insured still lives within
no longer had the right to rescind the contract of insurance as such period. After two years, the defenses of concealment or
rescission must allegedly be done during the lifetime of the misrepresentation, no matter how patent or well founded, no
insured within two years and prior to the commencement of longer lie. Congress felt this was a sufficient answer to the
action. various tactics employed by insurance companies to avoid
liability. The petitioners' interpretation would give rise to the
ISSUE: incongruous situation where the beneficiaries of an insured who
dies right after taking out and paying for a life insurance policy,
Whether the respondent insurer has the right to rescind would be allowed to collect on the policy even if the insured
the policy contract fraudulently concealed material facts.

RULING:
27. MANILA BANKERS LIFE INSURANCE CORPORATION
Yes. The so-called "incontestability clause" precludes vs ABAN
the insurer from raising the defenses of false representations or G.R. No. 175666. July 29, 2013
concealment of material facts insofar as health and previous
diseases are concerned if the insurance has been in force for at DOCTRINE:
least two years during the insured's lifetime. The phrase "during
the lifetime" found in Section 48 simply means that the policy is The ultimate aim of Section 48 of the Insurance Code is
no longer considered in force after the insured has died. The to compel insurers to solicit business from or provide insurance
key phrase in the second paragraph of Section 48 is "for a coverage only to legitimate and bona fide clients, by requiring
period of two years." them to thoroughly investigate those they insure within two
years from effectivity of the policy and while the insured is still
The policy was issued on November 6, 1973 and the alive. If they do not, they will be obligated to honor claims on the
insured died on April 26, 1975. The policy was thus in force for policies they issue, regardless of fraud, concealment or
a period of only one year and five months. Considering that the misrepresentation.
insured died before the two-year period had lapsed, respondent
company is not, therefore, barred from proving that the policy is FACTS:
void ab initio by reason of the insured's fraudulent concealment
or misrepresentation. Moreover, respondent company Delia Sotero (Sotero) took out a life insurance policy
rescinded the contract of insurance and refunded the premiums from Manila Bankers Life Insurance Corporation (Bankers Life),
paid on September 11, 1975, previous to the commencement designating respondent Cresencia P. Aban (Aban), her niece as
of this action on November 27, 1975. her beneficiary. Petitioner issued the policy on August 30, 1993,
after the requisite medical examination and payment of the RULING:
insurance premium. On April 10, 1996, when the insurance
policy had been in force for more than two years and seven Yes. It was Sotero who obtained the insurance for
months, Sotero died. Respondent filed a claim for the insurance herself. Allegations of fraud, which are predicated on
proceeds on July 9, 1996. respondent's alleged posing as Sotero and forgery of her
signature in the insurance application, are at once belied.
Petitioner denied respondent's claim on April 16, 1997 Fraudulent intent on the part of the insured must be established
and refunded the premiums paid on the policy. On April 24, to entitle the insurer to rescind the contract. In the absence of
1997, petitioner filed a civil case for rescission and/or annulment proof of such fraudulent intent, no right to rescind arises.
of the policy. It was alleged that the policy was obtained by
fraud, concealment and/or misrepresentation under the Section 48 serves a noble purpose, as it regulates the
Insurance Code, which thus renders it voidable under Article actions of both the insurer and the insured. Under the provision,
1390 of the Civil Code. Respondent filed a Motion to Dismiss an insurer is given two years — from the effectivity of a life
claiming that petitioner's cause of action was barred by insurance contract and while the insured is alive — to discover
prescription pursuant to Section 48 of the Insurance Code. RTC or prove that the policy is void ab initio or is rescindible by
granted the motion. Petitioner questioned the dismissal arguing reason of the fraudulent concealment or misrepresentation of
that the trial court erred in applying Section 48 and declaring the insured or his agent. It regulates both the actions of the
that prescription has set in. It contended that since it was insurers and prospective takers of life insurance. It gives
respondent — and not Sotero — who obtained the insurance, insurers enough time to inquire whether the policy was obtained
the policy issued was rendered void ab initio for want of by fraud, concealment, or misrepresentation; on the other hand,
insurable interest. The CA sustained the trial court and held that it forewarns scheming individuals that their attempts at
petitioner may no longer prove that the subject policy was void insurance fraud would be timely uncovered — thus deterring
ab initio or rescindable by reason of fraudulent concealment or them from venturing into such nefarious enterprise. At the same
misrepresentation after the lapse of more than two years from time, legitimate policy holders are absolutely protected from
its issuance. unwarranted denial of their claims or delay in the collection of
insurance proceeds occasioned by allegations of fraud,
Petitioner argued that Section 48 cannot apply to a case concealment, or misrepresentation by insurers, claims which
where the beneficiary under the insurance contract posed as may no longer be set up after the two-year period expires as
the insured and obtained the policy under fraudulent ordained under the law.
circumstances. It adds that respondent, who was merely
Sotero's niece, had no insurable interest in the life of her aunt. The said provision prevents a situation where the insurer
knowingly continues to accept annual premium payments on life
ISSUE: insurance, only to later on deny a claim on the policy on
specious claims of fraudulent concealment and
Whether petitioner company is liable to pay the claims misrepresentation, such as what obtains in the instant case.
Thus, instead of conducting at the first instance an investigation
into the circumstances surrounding the issuance of the policy in the task of supplying the information needed in the application.
this case which would have timely exposed the supposed flaws Respondent Ma. Celeste Abcede, Perla's daughter, signed the
and irregularities attending it as it now professes, petitioner application as sales counselor. Aside from pension benefits, the
appears to have turned a blind eye and opted instead to comprehensive pension plan also provided life insurance
continue collecting the premiums on the policy. For nearly three coverage to Florendo. This was covered by a Group Master
years, petitioner collected the premiums and devoted the same Policy that Philippine American Life Insurance Company
to its own profit. It cannot now deny the claim when it is called (Philam Life) issued to Philam Plans. If the plan holder died
to account. Section 48 must be applied to it with full force and before the maturity of the plan, his beneficiary was to instead
effect. receive the proceeds of the life insurance.

On October 30, 1997 Philam Plans issued Pension Plan


Agreement to Manuel, with petitioner Ma. Lourdes S. Florendo,
28. FLORENDO vs PHILAM PLANS, INC. his wife, as beneficiary. In time, Manuel paid his quarterly
G.R. No. 186983 February 22, 2012 premiums. Eleven months later or on September 15, 1998,
Manuel died of blood poisoning. Subsequently, Lourdes filed a
DOCTRINE: claim with Philam Plans for the payment of the benefits under
her husband's plan. Philam Plans declined the claim. Philam
It may be true that insured persons may accept policies Life found that Manuel was on maintenance medicine for his
without reading them, and that this is not negligence per heart and had an implanted pacemaker. Further, he suffered
se. But, this is not without any exception. It is and was from diabetes mellitus and was taking insulin. Lourdes
incumbent upon herein Manuel to read the insurance contracts, renewed her demand for payment under the plan but Philam
and this can be reasonably expected of him being a civil Plans rejected it, prompting her to filean action.
engineer and manager of a construction company, he is not
unschooled that the Court must come to his succor. In signing RTC rendered judgment, ordering Philam Plans, Perla
the pension plan application, he certified that he wrote all the and Ma. Celeste, solidarily, to pay Lourdes all the benefits from
information stated in it or had someone do it under his direction. her husband’s pension plan and the proceeds of his term
Assuming that it was Perla, the soliciting agent, who filled up the insurance, among others. It ruled that Manuel was not guilty of
application form, Manuel is still bound by what it contains since concealing the state of his health from his pension plan
he certified that he authorized her action. application. The CA reversed, holding that insurance policies
are traditionally contracts uberrimae fidae or contracts of utmost
FACTS: good faith. As such, it required Manuel to disclose to Philam
Plans conditions affecting the risk of which he was aware or
On October 23, 1997 Manuel Florendo filed an material facts that he knew or ought to know.
application for comprehensive pension plan with respondent
Philam Plans, Inc. after some convincing by private respondent Lourdes points out that, seeing the unfilled spaces in
Perla Abcede. Manuel signed the application and left to Perla Manuel’s pension plan application relating to his medical
history, Philam Plans should have returned it to him for Besides, Manuel had been taking medicine for his heart
completion. Since Philam Plans chose to approve the condition and diabetes when he submitted his pension plan
application just as it was, it cannot cry concealment on Manuel’s application. More, even if Perla’s knowledge of Manuel’s
part. Further, Lourdes adds that Philam Plans never queried pacemaker may be applied to Philam Plans under the theory of
Manuel directly regarding the state of his health. Consequently, imputed knowledge, it is not claimed that Perla was aware of his
it could not blame him for not mentioning it. two other afflictions that needed medical treatments. Pursuant
to Section 27 of the Insurance Code, Manuel’s concealment
ISSUE: entitles Philam Plans to rescind its contract of insurance with
him.
Whether Manuel is guilty of concealing his illness when
he kept blank and did not answer questions in his pension plan In signing the pension plan application, he certified that
application regarding the ailments he suffered from he wrote all the information stated in it or had someone do it
under his direction. Assuming that it was Perla who filled up the
RULING: application form, Manuel is still bound by what it contains since
he certified that he authorized her action. Philam Plans had
Yes. When Manuel signed the pension plan application, every right to act on the faith of that certification.
he adopted as his own the written representations and
declarations embodied in it. It is clear from these Lourdes could not seek comfort from her claim that
representations that he concealed his chronic heart ailment and Perla had assured Manuel that the state of his health would not
diabetes from Philam Plans. Since Manuel signed the hinder the approval of his application and that what is written on
application without filling in the details regarding his continuing his application made no difference to the insurance
treatments for heart condition and diabetes, the assumption is company. But, indubitably, Manuel was made aware when he
that he has never been treated for the said illnesses in the last signed the pension plan application that, in granting the same,
five years preceding his application. Philam Plans and Philam Life were acting on the truth of the
representations contained in that application. It may be true that
Lourdes insists that Manuel had concealed nothing insured persons may accept policies without reading them, and
since Perla, the soliciting agent, knew that Manuel had a that this is not negligence per se. But, this is not without any
pacemaker implanted on his chest in the 70s or about 20 years exception. It is and was incumbent upon Manuel to read the
before he signed up for the pension plan. But by its tenor, the insurance contracts, and this can be reasonably expected of
responsibility for preparing the application belonged to him being a civil engineer and manager of a construction
Manuel. Nothing in it implies that someone else may provide the company, he is not unschooled that the Court must come to his
information that Philam Plans needed. Manuel cannot sign the succor.
application and disown the responsibility for having it filled up. If
he furnished Perla the needed information and delegated to her Lastly, Lourdes points out that any defect or
the filling up of the application, then she acted on his instruction, insufficiency in the information provided by his pension plan
not on Philam Plan’s instruction. application should be deemed waived after the same has been
approved, the policy has been issued, and the premiums have
been collected. The Court cannot agree. The comprehensive on October 11, 1985, advising Tan's counsel that the Insurer's
pension plan that Philam Plans issued contains a one-year denial of Tan's claim remained unchanged.
incontestability period. The clause precludes the insurer from
disowning liability under the policy it issued on the ground of On November 20, 1985, Tan filed a civil case but
concealment or misrepresentation regarding the health of the petitioner filed a motion to dismiss on the alleged ground that
insured after a year of its issuance. Since Manuel died on the the action had already prescribed. Said motion was denied.
eleventh month following the issuance of his plan, the one year Petitioner went to CA and sought the nullification but was
incontestability period has not yet set in. Consequently, Philam denied.
Plans was not barred from questioning Lourdes entitlement to
the benefits of her husband’s pension plan. ISSUE:

Whether the filing of a motion for reconsideration


29. SUN INSURANCE vs. CA interrupts the stipulated 12 months prescriptive period to
G.R. No. 89741 March 13, 1991 contest the denial of the insurance claim
DOCTRINE: RULING:
The prescriptive period to file an action to the Insurance No. Condition of the Insurance Policy, provides that “if a
Commissioner or competent court after denial of claim claim be made and rejected and an action or suit be not
commences to run from the rejection of the demand. The commenced within twelve (12) months from receipt of notice of
pendency of the motion for reconsideration does not toll the such rejection xxx then the claim shall for all purposes be
running thereof. Otherwise, it can easily be used by insured deemed to have been abandoned and shall not thereafter be
persons as a scheme or device to waste time until any evidence recoverable.” Contracts of insurance, like other contracts, are to
which may be considered against them is destroyed. be construed according to the sense and meaning of the terms
which the parties themselves have used. If such terms are clear
FACTS: and unambiguous, they must be taken and understood in their
plain, ordinary and popular sense.
Emilio Tan took from herein petitioner property
insurance policy to cover his interest in the electrical supply The principle is laid down by this Court in the case of
store of his brother. 4 days after the issuance of the policy, the Ang v. Fulton Fire Insurance Co., to wit: "The condition
building was burned including the insured store. Tan filed his contained in an insurance policy that claims must be presented
claim for fire loss with petitioner, but on February 29, 1984, within one year after rejection is not merely a procedural
petitioner wrote Tan denying the latter's claim. On April 3, 1984, requirement but an important matter essential to a prompt
Tan wrote petitioner, seeking reconsideration of the denial of his settlement of claims against insurance companies as it
claim. On September 3, 1985, Tan's counsel wrote the Insurer demands that insurance suits be brought by the insured while
inquiring about the status of his April 3, 1984 request for
reconsideration. Petitioner answered the letter
the evidence as to the origin and cause of destruction have not FACTS: MSAS Cargo International Limited (MSAS) procured a
yet disappeared." marine insurance policy from respondent ICNA UK Limited of
London. The insurance was for a transshipment of certain
The contention of the respondents that the one-year wooden work tools and workbenches purchased for the
prescriptive period does not start to run until the petition for consignee Science Teaching Improvement Project (STIP),
reconsideration had been resolved by the insurer runs counter Ecotech Center, Sudlon Lahug, Cebu City, Philippines. ICNA
to the declared purpose. Moreover, it can easily be used by
issued an "all-risk" open marine policy. The cargo was shipped
insured persons as a scheme or device to waste time until any
"freight prepaid" from Hamburg, Germany on board M/S
evidence which may be considered against them is destroyed.
Katsuragi. A clean bill of lading was issued. The cargo was
received by petitioner Aboitiz Shipping Corporation
30. (Aboitiz).The bill of lading issued by Aboitiz contained the
notation "grounded outside warehouse." Upon arrival of the
cargo to Cebu City, the representative of the consignee school,
ABOITIZ SHIPPING CORPORATION vs INSURANCE Mr. Wilig, found that several work tools and workbenches were
COMPANY OF NORTH AMERICA found to have been completely soaked in water with most of the
packing cartons already disintegrating. Mr. Wilig immediately
G.R. No. 168402 August 6, 2008 contacted Aboitiz and ICNA.

DOCTRINE: The right of subrogation attaches upon payment The consignee contacted the Philippine office of ICNA for
by the insurer of the insurance claims by the assured. As insurance claims. ICNA paid the amount of P280,176.92 to
subrogee, the insurer steps into the shoes of the assured and consignee. A subrogation receipt was duly signed by Willig.
may exercise only those rights that the assured may have ICNA formally advised Aboitiz of the claim and subrogation
against the wrongdoer who caused the damage. receipt executed in its favor. Aboitiz refused to settle the claim.
Despite follow-ups, however, no reply was received from
Aboitiz. Aboitiz contends that ICNA has no cause of action
The giving of notice of loss or injury is a condition precedent to against it.
the action for loss or injury or the right to enforce the carrier's
liability. Circumstances peculiar to this case lead Us to conclude
that the notice requirement was complied with. ISSUE: Is respondent ICNA the real party-in-interest that
possesses the right of subrogation to claim reimbursement from
petitioner Aboitiz?
RULING: Yes. Respondent's cause of action is founded on it receiving payment from the insurer, the insured releases the
being subrogated to the rights of the consignee of the damaged wrongdoer who caused the loss from liability, the insurer loses
shipment. The right of subrogation springs from Article 2207 of its claim against the latter.
the Civil Code, which states:

The giving of notice of loss or injury is a condition precedent to


Article 2207. If the plaintiff's property has been insured, and he the action for loss or injury or the right to enforce the carrier's
has received indemnity from the insurance company for the liability. Circumstances peculiar to this case lead Us to conclude
injury or loss arising out of the wrong or breach of contract that the notice requirement was complied with. This notice
complained of, the insurance company shall be subrogated to requirement protects the carrier by affording it an opportunity to
the rights of the insured against the wrongdoer or the person make an investigation of the claim while the matter is still fresh
who has violated the contract. If the amount paid by the and easily investigated. It is meant to safeguard the carrier from
insurance company does not fully cover the injury or loss, the false and fraudulent claims.
aggrieved party shall be entitled to recover the deficiency from
the person causing the loss or injury.

Payment by the insurer to the assured operates as an equitable


assignment of all remedies the assured may have against the
third party who caused the damage. Subrogation is not
dependent upon, nor does it grow out of, any privity of contract
or upon written assignment of claim. It accrues simply upon
payment of the insurance claim by the insurer.

Upon payment to the consignee of indemnity for damage to the


insured goods, ICNA's entitlement to subrogation equipped it
with a cause of action against petitioner in case of a contractual
breach or negligence. This right of subrogation, however, has
its limitations. First, both the insurer and the consignee are
bound by the contractual stipulations under the bill of lading.
Second, the insurer can be subrogated only to the rights as the
insured may have against the wrongdoer. If by its own acts after
the subsequent delivery to its warehouses/plants. Glodel, in
turn, engaged the services of Loadmasters for the use of its
delivery trucks to transport the cargoes to Columbia’s
warehouses/plants in Bulacan and Valenzuela City. One truck
failed to deliver its cargo.

Columbia filed with R&B Insurance a claim for insurance


indemnity in the amount of ₱1,903,335.39. After the requisite
investigation and adjustment, R&B Insurance paid Columbia the
amount of ₱1,896,789.62 as insurance indemnity.

31.
R&B Insurance, thereafter, filed a complaint for damages
against both Loadmasters and Glodel. It sought reimbursement
of the amount it had paid to Columbia for the loss of the subject
LOADMASTERS CUSTOMS SERVICES, INC vs GLODEL
cargo. It claimed that it had been subrogated "to the right of the
BROKERAGE CORPORATION and R&B INSURANCE
consignee to recover from the party/parties who may be held
CORPORATION
legally liable for the loss.
G.R. No. 179446 January 10, 2011

ISSUE: Whether or not R&B Insurance’s contention is correct


DOCTRINE: As subrogee of the rights and interest of the that it had been surrogated to the rights of Columbia to file for
insured, the insurer has the right to seek reimbursement from damages against Loadmaster for the loss of its property
the party who caused the breach of contract and/or tort to the therefore holding the latter responsible for reimbursement to
damage of the insured. R&B Insurance.

FACTS: R&B Insurance issued a Marine Policy in favor of RULING: Yes. Subrogation is the substitution of one person in
Columbia to insure the shipment of 132 bundles of electric the place of another with reference to a lawful claim or right, so
copper cathodes against All Risks. The cargoes were shipped that he who is substituted succeeds to the rights of the other in
from Leyte to Manila. Columbia engaged the services of Glodel relation to a debt or claim, including its remedies or securities.
for the release and withdrawal of the cargoes from the pier and Doubtless, R&B Insurance is subrogated to the rights of the
insured to the extent of the amount it paid the consignee under
the marine insurance, as provided under Article 2207 of the Civil
Code, which reads:

ART. 2207. If the plaintiff’s property has been insured, and he


has received indemnity from the insurance company for the
injury or loss arising out of the wrong or breach of contract
complained of, the insurance company shall be subrogated to
the rights of the insured against the wrong-doer or the person
who has violated the contract. If the amount paid by the
insurance company does not fully cover the injury or loss, the
aggrieved party shall be entitled to recover the deficiency from
the person causing the loss or injury.

As subrogee of the rights and interest of the consignee, R&B


Insurance has the right to seek reimbursement from either
Loadmasters or Glodel or both for breach of contract and/or tort.
employer, Erlinda Fabie. Despite repeated demands,
defendants, failed and refused to pay the claim of PANMALAY.

PANMALAYAN filed a complaint for damages. Respondents


filed a motion to dismiss on the ground that petitioner has no
cause of action against them.

PANMALAYAN argued that its cause of action was anchored


upon Article 2207 of the Civil Code, , which reads:
32.

If the plaintiffs property has been insured, and he has received


PAN MALAYAN INSURANCE CORPORATION vs CA indemnity from the insurance company for the injury or loss
G.R. No. 81026 April 3, 1990 arising out of the wrong or breach of contract complained of, the
insurance company shall be subrogated to the rights of the
insured against the wrongdoer or the person who has violated
the contract.
DOCTRINE: The right of subrogation is not dependent upon,
nor does it grow out of, any privity of contract or upon written
assignment of claim. It accrues simply upon payment of the
insurance claim by the insurer ISSUE: Whether or not the insurer PANMALAY may institute an
action to recover the amount it had paid its assured in
settlement of an insurance claim against private respondents as
the parties allegedly responsible for the damage caused to the
FACTS: Petitioner Pan Malayan Insurance Company
insured vehicle.
(PANMALAYAN) nsured a Mitsubishi Colt Lancer registered in
the name of Canlubang Automotive Resources Corporation
(Canlubang). On May 26, 1985, due to the "carelessness,
recklessness, and imprudence" of the unknown driver of a pick- RULING: Yes. Article 2207 of the Civil Code is founded on the
up, the insured car was hit and suffered damages in the amount well-settled principle of subrogation. If the insured property is
of P42,052.00. PANMALAY defrayed the cost of repair of the destroyed or damaged through the fault or negligence of a party
insured car and, therefore, was subrogated to the rights of other than the assured, then the insurer, upon payment to the
CANLUBANG against the driver of the pick-up and his assured, will be subrogated to the rights of the assured to
recover from the wrongdoer to the extent that the insurer has
been obligated to pay. Payment by the insurer to the assured
operates as an equitable assignment to the former of all
remedies which the latter may have against the third party
whose negligence or wrongful act caused the loss. The right of
subrogation is not dependent upon, nor does it grow out of, any
privity of contract or upon written assignment of claim. It accrues
simply upon payment of the insurance claim by the insurer.

There are a few recognized exceptions to this rule. For instance,


if the assured by his own act releases the wrongdoer or third
party liable for the loss or damage, from liability, the insurer's
right of subrogation is defeated Similarly, where the insurer
pays the assured the value of the lost goods without notifying
the carrier who has in good faith settled the assured's claim for
loss, the settlement is binding on both the assured and the
insurer, and the latter cannot bring an action against the carrier
on his right of subrogation. And where the insurer pays the
assured for a loss which is not a risk covered by the policy,
thereby effecting "voluntary payment", the former has no right
of subrogation against the third party liable for the loss.

None of the exceptions are availing in the present case.


Nestle subsequently filed a claim against E. Razon, Mabuhay,
the Port Authority, and its insurer, the Home Insurance
Corporation, for P49,170.00. When the other companies denied
liability, Home Insurance paid the claim and was issued a
subrogation receipt for $6,070.00.

Mabuhay alone was sued by Home Insurance for the recovery


of the amount it had paid to Nestle. Mabuhay denied liability.
33.

After trial, the Regional Trial Court of Manila rendered judgment


HOME INSURANCE CORPORATION vs CA dismissing the complaint declaring that the plaintiff failed to
establish the legal and factual bases for its claim. The decision
G.R. No. 109293 August 18, 1993 noted that the insurance contract between the corporation and
the consignee was not presented and that the other supporting
documents were all only photocopies. No explanation was given
DOCTRINE: The right of subrogation must be supported by the for the failure of the plaintiffs to submit the originals.
insurance contract itself. A subrogation receipt is not sufficient
to prove the petitioner's claim holding the respondent liable for
damages. Home Insurance argued that the subrogation receipt proves the
existence of the insurance contract between Nestle and the
Home Insurance and the amount paid by the latter to the former.
FACTS: Filipro Phil. now known as Nestle Phil., was the
consignee of two hydraulic engines shipped by INREDECO
from the United States on the M/S Oriental Satesman. It was ISSUE: Whether or not a subrogation receipt proves the
turned over to E. Razon Arrastre, which retained its custody. existence of the insurance contract.
The cargo was later hauled by Mabuhay Brokerage Co. to its
warehouse. The cargoes were eventually delivered to the
consignee. It was later discovered that one of the engines was RULING: No. By itself alone, the subrogation receipt is not
damaged. sufficient to prove the petitioner's claim holding the respondent
liable for the damage to the engine.
As a mere subrogee of Nestle, Home can exercise only such
rights against the parties handling the cargo as were granted to
Nestle under the insurance contract. The insurance contract
would have clearly indicated the scope of the coverage but
there is no evidence of this. It cannot simply be supposed that
the hauling was included in the coverage; it is possible that the
coverage ended with the arrastre. In other words, then rights
transferred to Home by Nestle — still assuming there was a
valid subrogation — might not include the right to sue Mabuhay.

The insurance contract might have proved that it covered the


hauling portion of the shipment and was not limited to the
transport of the cargo while at sea, if that were really the case.
It could have shown that the agreement was not only a marine
transportation insurance but covered all phases of the cargo's
shipment, from the time the cargo was loaded on the vessel in
the United States until it was delivered to the consignee in the
Philippines. But there is no acceptable evidence of these
stipulations because the original contract of insurance has not
been presented.
date of filing of the complaint up to and until the same is fully
paid, and 25% of the claim as attorney's fees.

The trial court ruled that private respondent Prudential failed to


prove by clear, convincing, and competent evidence that there
was a shortage in the shipment. The trial court said that private
respondent Prudential failed to establish by competent
evidence the genuineness and due execution of the bill of lading
34. and the true and exact weight of the shipment when it was
loaded unto the vessel. Accordingly, the trial court dismissed
both the complaint and the counterclaim. On appeal, the Court
of Appeals reversed the decision of the trial court. Hence, the
WALLEM PHILIPPINES SHIPPING INC. and SEACOAST
present petition.
MARITIME CORPORATION vs PRUDENTIAL GUARANTEE
& ASSURANCE INC.
G.R. No. 152158 February 7, 2003 ISSUE: Whether or not Prudential can rightfully claim damages
against Wallem despite the non-presentation of the insurance
contract.
DOCTRINE: The contract of insurance must be presented in
evidence to indicate its coverage.
RULING: No. Even if the shortage can be definitively
determined, Wallem still cannot be held liable because of the
FACTS: Private respondent Prudential Guarantee & Assurance failure of Prudential to present the contract of insurance or a
Inc. (Prudential) brought an action copy thereof. Prudential claims that it is subrogated to the rights
of GMC pursuant to their insurance contract. For this purpose,
for damages and attorney's fees against Wallem Philippines it submitted a subrogation receipt (Exh. J) and a marine cargo
Shipping, Inc. (Wallem) and Seacoast Maritime Corporation risk note (Exh. D). However, as the trial court pointed out, this
(Seacoast). Prudential sought the recovery of the sum of is not sufficient. As GMC's subrogee, Prudential can exercise
P995,677.00, representing the amount it had paid to its insured, only
General Milling Corporation (GMC), for alleged shortage
incurred in the shipment of "Indian Toasted Soyabean those rights granted to GMC under the insurance contract. The
Extraction Meal, Yellow," with 6% legal interest thereon from the contract of insurance must be presented in evidence to indicate
the extent of its coverage.
stevedoring services, unloaded the 60,000 bags of soda ash
dense from the vessel and brought them to the open storage
area of petitioner for temporary storage and safekeeping,
pending clearance from the Bureau of Customs and delivery to
the consignee.When the unloading of the bags was completed,
2,702 bags were found to be in bad order condition. After all the
bags were unloaded in the warehouses of the consignee, a total
of 2,881 bags were in bad order condition due to spillage,
caking, and hardening of the contents.

35. Respondent, as insurer, paid the value of the lost/ damaged


cargoes to the consignee in the amount of ₱643,600.25.

ASIAN TERMINALS, INC. vs MALAYAN INSURANCE, CO.,


INC. Respondent, as subrogee of the consignee, filed before the
Regional Trial Court (RTC) of Manila, Branch 35, a Complaint
G.R. No. 171406 April 4, 2011
for damages against petitioner, the shipper Inchcape Shipping
Services, and the cargo broker MEC Customs Brokerage. Trial
ensued. The RTC rendered a decision in favor of respondent.
DOCTRINE: On appeal, the CA affirmed the RTC’s decision.

FACTS: Shandong Weifang Soda Ash Plant shipped on board Petitioner contends that respondent has no cause of action
the vessel MV "Jinlian I" 60,000 plastic bags of soda ash dense because it failed to present the insurance contract or policy
(each bag weighing 50 kilograms) from China to Manila. The covering the subject shipment. Petitioner argues that the
shipment was insured with respondent Malayan Insurance Subrogation Receipt presented by respondent is not sufficient
Company, Inc. to prove that the subject shipment was insured and that
respondent was validly subrogated to the rights of the
consignee.Thus, petitioner submits that without proof of a valid
Upon arrival of the vessel at Pier 9, South Harbor, Manila, the subrogation, respondent is not entitled to any reimbursement.
stevedores of petitioner Asian Terminals, Inc., a duly registered
domestic corporation engaged in providing arrastre and
ISSUE: Whether the non-presentation of the insurance contract liability, the insurer’s right of subrogation is defeated. Similarly,
or policy is fatal to respondent’s cause of action where the insurer pays the assured the value of the lost goods
without notifying the carrier who has in good faith settled the
assured’s claim for loss, the settlement is binding on both the
assured and the insurer, and the latter cannot bring an action
RULING: No. The general rule that the non-presentation of the
against the carrier on his right of subrogation. And where the
policy is fatal to the subrogee’s cause is not applicable in this insurer pays the assured for a loss which is not a risk covered
case. by the policy, thereby effecting ‘voluntary payment,’ the former
has no right of subrogation against the third party liable for the
loss.
Here, the loss of the cargo undoubtedly occured while in the
custody of the carrier and during the unloading thereof by the FACTS:
stevedores of Asian Terminal. The presentation in evidence of
the marine insurance policy is not indispensable before the Loadstar Shipping and Philippine Associated Smelting
insurer may recover from the common carrier the insured value and Refining Corporation (PASAR) entered into a Contract of
of the lost cargo in the exercise of its subgrogatory right. The Affreightment for domestic bulk transport of the latter’s copper
subrogation receipt, by itself, is sufficient to establish not only concentrates for a period of one year from November 1, 1998
the relationship of herein private respondent as insurer and to October 31, 1999. The contract was extended up to the end
Shandong, as the assured shipper of the lost cargo of industrial of October 2000. On September 10, 2000, 5,065.47 wet metric
tons (WMT) of copper concentrates were loaded in Cargo Hold
fuel oil, but also the amount paid to settle the insurance claim.
Nos. 1 and 2 of MV "Bobcat", a marine vessel owned by
The right of subrogation accrues simply upon payment by the
Loadstar International and operated by Loadstar Shipping
insurance company of the insurance claim. under a charter party agreement. The shipper and consignee
36.) LOADSTAR SHIPPING COMPANY, INC. V. MALAYAN under the Bill of Lading are Philex Mining Corporation (Philex)
and PASAR, respectively. The cargo was insured with Malayan
INSURANCE COMPANY, INC. 742 SCRA 627
Insurance Comoany. P & I Association is the third party liability
insurer of Loadstar Shipping.
DOCTRINE:
On said date (September 10, 2000), MV "Bobcat" sailed
The right of subrogation is not dependent upon, nor from La Union bound for Isabel, Leyte. On September 12, 2000,
does it grow out of, any privity of contract or upon written while in the vicinity of Cresta de Gallo, the vessel’s chief officer
assignment of claim. It accrues simply upon payment of the on routine inspection found a crack on starboard sideof the main
insurance claim by the insurer. The right of subrogation is deck which caused seawater to enter and wet the cargo inside
however, not absolute. There are a few recognized exceptions Cargo Hold No. 2 forward/aft. Elite Surveyor recommended
to this rule. For instance, if the assured by his own act releases payment to the assured the amount of [P]32,351,102.32 as
the wrongdoer or third party liable for the loss or damage, from
adjusted. On the basis of such recommendation, Malayan paid damages, less US$90,000.00, plus legal interest at 6%
PASAR the amount of [P]32,351,102.32. annually. from the date of the trial court’s decision. Upon the
finality of the decision, the total amount of the judgment shall
Meanwhile, on November 24, 2000, Malayan wrote earn annual interest at 12% until full payment.
Loadstar Shipping informing the latter of a prospective buyer for
the damaged copper concentrates and the opportunity to ISSUE:
nominate/refer other salvage buyers to PASAR. On November
29, 2000, Malayan wrote Loadstar Shipping informing the latter W/n respondent’s payment to PASAR, on the basis of the
of the acceptance of PASAR’s proposal to take the damaged latter’s fraudulent claim, entitled respondent automatic right of
copper concentrates at a residual value of US$90,000.00. On recovery by virtue of subrogation.
December 9, 2000, Loadstar Shipping wrote Malayan
requesting for the reversal of its decision to accept PASAR’s HELD:
proposal and the conduct of a public bidding to allow Loadstar
Shipping to match or top PASAR’s bid by 10%. NO. The right of subrogation stems from Article 2207.

On January 23, 2001, PASAR signed a subrogation "The right of subrogation is not dependent upon, nor
receipt in favor of Malayan. To recover the amount paid and in does it grow out of, any privity of contract or upon written
the exercise of its right of subrogation, Malayan demanded assignment of claim. It accrues simply upon payment of the
reimbursement from Loadstar Shipping, which refused to insurance claim by the insurer." The right of subrogation is
comply. Consequently, on September 19, 2001, Malayan however, not absolute. "There are a few recognized exceptions
instituted with the RTC a complaint for damages. to this rule. For instance, if the assured by his own act releases
the wrongdoer or third party liable for the loss or damage, from
RTC dismissed the complaint. It was convinced that the vessel liability, the insurer’s right of subrogation is defeated. x x x
was seaworthy at the time of loading and that the damage was Similarly, where the insurer pays the assured the value of the
attributable to the perils of the sea (natural disaster) and not due lost goods without notifying the carrier who has in good faith
to the fault or negligence of Loadstar Shipping. The RTC found settled the assured’s claim for loss, the settlement is binding on
that although contaminated by seawater, the copper both the assured and the insurer, and the latter cannot bring an
concentrates can still be used. It gave credence to the testimony action against the carrier on his right of subrogation. x x x And
of Francisco Esguerra, defendants-appellees’ expert witness, where the insurer pays the assured for a loss which is not a risk
that despite high chlorine content, the copper concentrates covered by the policy, thereby effecting ‘voluntary payment,’ the
remain intact and will not lose their value. The gold and silver former has no right of subrogation against the third party liable
remain with the grains/concentrates even if soaked with for the loss x x x."
seawater and does not melt.
The rights of a subrogee cannot be superior to the rights
CA granted the appeal of Malayan ordering defendants- possessed by a subrogor. "Subrogation is the substitution of
appellees to pay plaintiff-appellant ₱33,934,948.75 as actual one person in the place of another with reference to a lawful
claim or right, so that he who is substituted succeeds to the cause of action is nil. As Malayan is claiming for actual
rights of the other in relation to a debt or claim, including its damages, it bears the burden of proof to substantiate its claim.
remedies or securities. The rights to which the subrogee
succeeds are the same as, but not greater than, those of the "The burden of proof is on the party who would be
person for whom he is substituted, that is, he cannot acquire defeated if no evidence would be presented on either side. The
any claim, security or remedy the subrogor did not have. In other burden is to establish one’s case by a preponderance of
words, a subrogee cannot succeed to a right not possessed by evidence which means that the evidence, as a whole, adduced
the subrogor. by one side, is superior to that of the other. Actual damages are
not presumed. The claimant must prove the actual amount of
A subrogee in effect steps into the shoes of the insured loss with a reasonable degree of certainty premised upon
and can recover only if the insured likewise could have competent proof and on the best evidence obtainable. Specific
recovered." Consequently, an insurer indemnifies the insured facts that could afford a basis for measuring whatever
based on the loss or injury the latter actually suffered from. If compensatory or actual damages are borne must be pointed
there is no loss or injury, then there is no obligation on the part out. Actual damages cannot be anchored on mere surmises,
of the insurer to indemnify the insured. Should the insurer pay speculations or conjectures."
the insured and it turns out that indemnification is not due, or if
due, the amount paid is excessive, the insurer takes the risk of Having ruled that Malayan did not adduce proof of
not being able to seek recompense from the alleged wrongdoer. pecuniary loss to PASAR for which the latter was questionably
This is because the supposed subrogor did not possess the indemnified, there is no necessity to expound further on the
right to be indemnified and therefore, no right to collect is other issues raised by the petitioners and Malayan in this case.
passed on to the subrogee. As regards the determination of
actual damages, "[i]t is axiomatic that actual damages must be 37.) NEW WORLD INTERNATIONAL DEVELOPMENT
proved with reasonable degree of certainty and a party is PHILS. , INC. V. NYK-FILJAPAN SHIPPING CORP., 656
entitled only to such compensation for the pecuniary loss that
SCRA 129, 24 AUGUST 2011
was duly proven." Article 2199 of the New Civil Code speaks of
how actual damages are awarded:
DOCTRINE:
Art. 2199. Except as provided by law or by stipulation,
one is entitled to an adequate compensation only for such An all-risk policy insure against all causes of
pecuniary loss suffered by him as he has duly proved. Such conceivable loss or damage except when otherwise excluded
compensation is referred to as actual or compensatory or when the loss or damage was due to fraud or intentional
damages. misconduct committed by the insured. The policy covered all
losses during the voyage whether or not arising from a marine
Malayan never presented evidence of its own in peril.
refutation to Esguerra’s testimony. And, even if the Court will
disregard the entirety of his testimony, the effect on Malayan’s FACTS:
Petitioner New World bought from DMT Corporation New World demanded recompense for its loss from
(DMT) through its agent, Advatech three emergency generator respondents NYK, DMT, Advatech, LEP Profit, LEP
sets worth US$721,500.00. DMT shipped the generator sets by International Philippines, Inc. (LEP), Marina, and Serbros. While
truck from Wisconsin, United States, to LEP Profit in Chicago, LEP and NYK acknowledged receipt of the demand, both
Illinois. From there, the shipment went by train to Oakland, denied liability for the loss.
California, where it was loaded on S/S California Luna V59,
owned and operated by NYK for delivery to petitioner New RTC rendered a decision absolving the various respondents
World in Manila. NYK issued a bill of lading, declaring that it from liability with the exception of NYK. The RTC found that the
received the goods in good condition. NYK unloaded the generator sets were damaged during transit while in the care of
shipment in Hong Kong and transshipped it to S/S ACX Ruby NYK’s vessel, ACX Ruby. The latter failed, according to the
V/72 that it also owned and operated. RTC, to exercise the degree of diligence required of it in the face
of a foretold raging typhoon in its path.
On its journey to Manila, however, ACX Ruby
encountered typhoon Kadiang whose captain filed a sea protest CA affirmed the RTC’s rulings except with respect to
on arrival at the Manila South Harbor on October 5, 1993 Seaboard’s liability. The CA held that petitioner New World can
respecting the loss and damage that the goods on board his still recoup its loss from Seaboard’s marine insurance policy,
vessel suffered. Marina Port Services, Inc. (Marina), the Manila considering a) that the submission of the itemized listing is an
South Harbor arrastre or cargo-handling operator, received the unreasonable imposition and b) that the one-year prescriptive
shipment on October 7, 1993. Upon inspection of the three period under the COGSA did not affect New World’s right under
container vans separately carrying the generator sets, two vans the insurance policy since it was the Insurance Code that
bore signs of external damage while the third van appeared governed the relation between the insurer and the insured.
unscathed.
ISSUE:
The shipment remained at Pier 3s Container Yard under
Marinas care pending clearance from the Bureau of Customs. W/N New World is entitled to the claim under the policy.
Eventually, on October 20, 1993 customs authorities allowed
petitioners customs broker, Serbros Carrier Corporation HELD:
(Serbros), to withdraw the shipment and deliver the same to
petitioner New Worlds job site in Makati City. Yes. The Court does not regard as substantial the
question of reasonableness of Seaboard’s additional
An examination of the three generator sets in the requirement of an itemized listing of the damage that the
presence of petitioner New Worlds representatives, Federal generator sets suffered. The record shows that petitioner New
Builders (the project contractor) and surveyors of petitioner New World complied with the documentary requirements evidencing
Worlds insurer, SeaboardEastern Insurance Company damage to its generator sets.
(Seaboard), revealed that all three sets suffered extensive
damage and could no longer be repaired. For these reasons,
The marine open policy that Seaboard issued to New For a vessel to be seaworthy, it must be adequately
World was an all-risk policy. Such a policy insured against all equipped for the voyage and manned with a sufficient number
causes of conceivable loss or damage except when otherwise of competent officers and crew.
excluded or when the loss or damage was due to fraud or
intentional misconduct committed by the insured. The policy
covered all losses during the voyage whether or not arising from FACTS:
a marine peril.
On December 19, 1987, motor tanker MT Vector left Limay,
Here, the policy enumerated certain exceptions like Bataan, at about 8:00 p.m., enroute to Masbate, loaded with
unsuitable packaging, inherent vice, delay in voyage, or vessels 8,800 barrels of petroleum products shipped by petitioner
unseaworthiness, among others. But Seaboard had been Caltex. MT Vector is a tramping motor tanker owned and
unable to show that petitioner New World’s loss or damage fell operated by Vector Shipping Corporation, engaged in the
within some or one of the enumerated exceptions. business of transporting fuel products such as gasoline,
kerosene, diesel and crude oil. During that particular voyage,
the MT Vector carried on board gasoline and other oil products
What is more, Seaboard had been unable to explain
owned by Caltex by virtue of a charter contract between them.
how it could not verify the damage that New World’s goods
suffered going by the documents that it already submitted. On December 20, 1987, at about 6:30 a.m., the passenger
Seaboard cannot pretend that the above documents are ship MV Doa Paz left the port of Tacloban headed for Manila
inadequate since they were precisely the documents listed in its with a complement of 59 crew members including the master
insurance policy. Being a contract of adhesion, an insurance and his officers, and passengers totaling 1,493 as indicated in
policy is construed strongly against the insurer who prepared it. the Coast Guard Clearance. The MV Doa Paz is a passenger
The Court cannot read a requirement in the policy that was not and cargo vessel owned and operated by Sulpicio Lines, Inc.
there. plying the route of Manila/ Tacloban/ Catbalogan/ Manila/
Catbalogan/ Tacloban/ Manila, making trips twice a week.
Further, it appears from the exchanges of
At about 10:30 p.m. of December 20, 1987, the two vessels
communications between Seaboard and Advatech that
collided in the open sea within the vicinity of Dumali Point
submission of the requested itemized listing was incumbent on
between Marinduque and Oriental Mindoro. All the
the latter as the seller DMT’s local agent. Petitioner New World
crewmembers of MV Doa Paz died, while the two survivors from
should not be made to suffer for Advatech’s shortcomings.
MT Vector claimed that they were sleeping at the time of the
incident.
38.) CALTEX PHILS., INC. V. SULPICIO LINES, 315 SCRA
709, 30 SEPTEMBER 1999 The MV Doa Paz carried an estimated 4,000 passengers;
many indeed, were not in the passenger manifest. Only 24
DOCTRINE: survived the tragedy after having been rescued from the burning
waters by vessels that responded to distress calls. Among those
who perished were public school teacher Sebastian Caezal (47
years old) and his daughter Corazon Caezal (11 years old), both Under the Carriage of Goods by Sea Act :
unmanifested passengers but proved to be on board the vessel. Sec. 3. (1) The carrier shall be bound before and at the
beginning of the voyage to exercise due diligence to -
On March 22, 1988, the board of marine inquiry in BMI (a) Make the ship seaworthy;
Case No. 653-87 after investigation found that the MT Vector, (b) Properly man, equip, and supply the ship;
its registered operator Francisco Soriano, and its owner and xxx xxx xxx
actual operator Vector Shipping Corporation, were at fault and
responsible for its collision with MV Doa Paz. A complaint for Thus, the carriers are deemed to warrant impliedly the
Damages Arising from Breach of Contract of Carriage against seaworthiness of the ship. For a vessel to be seaworthy, it
Sulpicio Lines, Inc. must be adequately equipped for the voyage and manned
with a sufficient number of competent officers and
crew. The failure of a common carrier to maintain in seaworthy
RTC ruled against defendant-3rd party plaintiff Sulpicio Lines, condition the vessel involved in its contract of carriage is a clear
Inc. breach of its duty prescribed in Article 1755 of the Civil Code.
The provisions owed their conception to the nature of the
business of common carriers. This business is impressed with
On appeal to the CA interposed by Sulpicio Lines, Inc., it a special public duty. The public must of necessity rely on the
modified the trial court’s ruling and included petitioner Caltex as care and skill of common carriers in the vigilance over the goods
one of those liable for damages. and safety of the passengers, especially because with the
modern development of science and invention, transportation
Third party defendants Vector Shipping Co. and Caltex has become more rapid, more complicated and somehow more
(Phils.), Inc. are held equally liable under the third party hazardous. For these reasons, a passenger or a shipper of
complaint to reimburse/indemnify defendant Sulpicio Lines, Inc. goods is under no obligation to conduct an inspection of the ship
of the above-mentioned damages, attorney’s fees and costs and its crew, the carrier being obliged by law to impliedly
which the latter is adjudged to pay plaintiffs, the same to be warrant its seaworthiness.
shared half by Vector Shipping Co. (being the vessel at fault for
the collision) and the other half by Caltex (Phils.), Inc. (being the
charterer that negligently caused the shipping of combustible
cargo aboard an unseaworthy vessel). 39.) ROQUE V. INTERMEDIATE APPELLATE COURT, 139
SCRA 596, 11 NOVEMBER 1985
ISSUE:
DOCTRINE:
When is a vessel considered seaworthy?
Since the law provides for an implied warranty of
seaworthiness in every contract of ordinary marine insurance, it
HELD: becomes the obligation of a cargo owner to look for a reliable
common carrier which keeps its vessels in seaworthy condition.
The shipper of cargo may have no control over the vessel but further found that one of the hatches was left open causing
he has full control in the choice of the common carrier that will water to enter the barge and because the barge was not
transport his goods. Or the cargo owner may enter into a provided with the necessary cover or tarpaulin, the ordinary
contract of insurance which specifically provides that the insurer splash of sea waves brought more water inside the barge.
answers not only for the perils of the sea but also provides for
coverage of perils of the ship. On March 8, 1972, the petitioners wrote a letter to Manila
Bay demanding payment of P150,000.00 for the loss of the
Barratry as defined in American Insurance Law is "any shipment plus P100,000.00 as unrealized profits but the latter
willful misconduct on the part of master or crew in pursuance of ignored the demand. Another letter was sent to respondent
some unlawful or fraudulent purpose without the consent of the Pioneer claiming the full amount of P100,000.00 under the
owners, and to the prejudice of the owner's interest." insurance policy but respondent refused to pay on the ground
that its hability depended upon the "Total loss by Total Loss of
Barratry necessarily requires a willful and intentional act Vessel only". Hence, petitioners commenced Civil Case No.
in its commission. No honest error of judgment or mere 86599 against Manila Bay and respondent Pioneer.
negligence, unless criminally gross, can be barratry.
The Trial Court ordered condemning defendants Manila Bay
FACTS: Lighterage Corporation and Pioneer Insurance and Surety
Corporation to pay plaintiffs, jointly and severally, the sum of
P100,000.00;
On February 19, 1972, the Manila Bay Lighterage
Corporation (Manila Bay), a common carrier, entered into a
contract with the petitioners whereby the former would load and The Appellate Court modified the trial court's decision and
carry on board its barge Mable 10 about 422.18 cubic meters of absolved Pioneer from liability after finding that there was a
logs from Malampaya Sound, Palawan to North Harbor, Manila. breach of implied warranty of seaworthiness on the part of the
The petitioners insured the logs against loss for P100,000.00 petitioners and that the loss of the insured cargo was caused by
with respondent Pioneer Insurance and Surety Corporation the "perils of the ship" and not by the "perils of the sea". It ruled
(Pioneer). that the loss is not covered by the marine insurance policy.

On February 29, 1972, the petitioners loaded on the ISSUES:


barge, 811 pieces of logs at Malampaya Sound, Palawan for
1.) In Marine Cargo Insurance, is there warranty of
carriage and delivery to North Harbor, Port of Manila, but the
shipment never reached its destination because Mable 10 sank seaworthiness by the cargo owner?
with the 811 pieces of logs somewhere off Cabuli Point in
Palawan on its way to Manila. As alleged by the petitioners in 2.) Can petitioners allege barratry on the basis of the findings
their complaint and as found by both the trial and appellate showing negligence on the part of the vessel's crew?
courts, the barge where the logs were loaded was not
seaworthy such that it developed a leak. The appellate court HELD:
1.) Yes. The liability of the insurance company is governed Since the law provides for an implied warranty of
by law. Section 113 of the Insurance Code provides: seaworthiness in every contract of ordinary marine insurance, it
becomes the obligation of a cargo owner to look for a reliable
In every marine insurance upon a ship or freight, or common carrier which keeps its vessels in seaworthy condition.
freightage, or upon any thing which is the subject of The shipper of cargo may have no control over the vessel but
marine insurance, a warranty is implied that the ship is he has full control in the choice of the common carrier that will
seaworthy. transport his goods. Or the cargo owner may enter into a
contract of insurance which specifically provides that the insurer
Section 99 of the same Code also provides in part. answers not only for the perils of the sea but also provides for
coverage of perils of the ship.
Marine insurance includes:
2.) NO. Barratry as defined in American Insurance Law is
(1) Insurance against loss of or damage to: "any willful misconduct on the part of master or crew in
pursuance of some unlawful or fraudulent purpose
(a) Vessels, craft, aircraft, vehicles, goods, without the consent of the owners, and to the prejudice
freights, cargoes, merchandise, ... of the owner's interest."

From the above-quoted provisions, there can be no Barratry necessarily requires a willful and intentional act in its
mistaking the fact that the term "cargo" can be the subject of commission. No honest error of judgment or mere negligence,
marine insurance and that once it is so made, the implied unless criminally gross, can be barratry.
warranty of seaworthiness immediately attaches to whoever is
insuring the cargo whether he be the shipowner or not. Here, there is no finding that the loss was occasioned by the
willful or fraudulent acts of the vessel's crew. There was only
In the case of Go Tiaoco y Hermanos v. Union simple negligence or lack of skill.
Insurance Society of Canton (40 Phil. 40):
40.) ORIENTAL ASSURANCE CORPORATION V. CA, 200
The same conclusion must be reached if the SCRA 459, 9 AUGUST 1991
question be discussed with reference to the
seaworthiness of the ship. It is universally DOCTRINE:
accepted that in every contract of insurance
upon anything which is the subject of marine A constructive total loss is one which gives to a person
insurance, a warranty is implied that the ship insured a right to abandon. There is constructive total loss if
shall be seaworthy at the time of the inception of more than three-fourths thereof in value is actually lost, or
the voyage. This rule is accepted in our own would have to be expended to recover it from the peril.
Insurance Law.
FACTS: Respondent Appellate Court affirmed the lower Court
judgment in all respects except for the rate of interest, which
Sometime in January 1986, private respondent Panama was reduce from twelve (12%) to six (6%) per annum.
Sawmill Co., Inc. (Panama) bought, in Palawan, 1,208 pieces
of apitong logs, with a total volume of 2,000 cubic meters. It Both Courts shared the view that the insurance contract should
hired Transpacific Towage, Inc., to transport the logs by sea to be liberally construed in order to avoid a denial of substantial
Manila and insured it against loss for P1-M with petitioner justice; and that the logs loaded in the two barges should be
Oriental Assurance Corporation (Oriental Assurance). There is treated separately such that the loss sustained by the shipment
a claim by Panama, however, that the insurance coverage in one of them may be considered as "constructive total loss"
should have been for P3-M were it not for the fraudulent act of and correspondingly compensable.
one Benito Sy Yee Long to whom it had entrusted the amount
of P6,000.00 for the payment of the premium for a P3-M policy. ISSUE:

On 28 January 1986, the two barges were towed by one Whether or not Oriental Assurance can be held liable under its
tug-boat, the MT 'Seminole' But, as fate would have it, during marine insurance policy based on the theory of a divisible
the voyage, rough seas and strong winds caused damage to contract of insurance and, consequently, a constructive total
Barge TPAC-1000 resulting in the loss of 497 pieces of logs out loss.
of the 598 pieces loaded thereon.
HELD:
Panama demanded payment for the loss but Oriental
Assurance refuse on the ground that its contracted liability was No liability attaches. The terms of the contract constitute
for "TOTAL LOSS ONLY." The rejection was upon the the measure of the insurer liability and compliance therewith is
recommendation of the Tan Gatue Adjustment Company. a condition precedent to the insured's right to recovery from the
insurer. Whether a contract is entire or severable is a question
Unable to convince Oriental Assurance to pay its claim, of intention to be determined by the language employed by the
Panama filed a Complaint for Damages against Ever Insurance parties. The policy in question shows that the subject matter
Agency (allegedly, also liable), Benito Sy Lee Yong and Oriental insured was the entire shipment of 2,000 cubic meters of
Assurance, before the Regional Trial Court, Kalookan, Branch apitong logs. The fact that the logs were loaded on two different
123, docketed as Civil Case No. C-12601. barges did not make the contract several and divisible as to the
items insured. The logs on the two barges were not separately
The RTC rendered its decision ordering the defendant Oriental valued or separately insured. Only one premium was paid for
Assurance Corporation to pay plaintiff Panama Saw Mill Inc. the the entire shipment, making for only one cause or consideration.
amount of P415,000.00 as insurance indemnity with interest at The insurance contract must, therefore, be considered
the rate of 12% per annum. indivisible.
More importantly, the insurer's liability was for "total loss The requirements for the application of Section 139 of
only." A total loss may be either actual or constructive (Sec. 129, the Insurance Code, quoted above, have not been met. The
Insurance Code). An actual total loss is caused by: logs involved, although placed in two barges, were not
separately valued by the policy, nor separately insured.
(a) A total destruction of the thing insured; Resultantly, the logs lost in barge TPAC-1000 in relation to the
total number of logs loaded on the same barge cannot be made
(b) The irretrievable loss of the thing by sinking, or by the basis for determining constructive total loss. The logs having
being broken up; been insured as one inseparable unit, the correct basis for
determining the existence of constructive total loss is the totality
(c) Any damage to the thing which renders it valueless of the shipment of logs. Of the entirety of 1,208, pieces of logs,
to the owner for the purpose for which he held it; or only 497 pieces thereof were lost or 41.45% of the entire
shipment. Since the cost of those 497 pieces does not exceed
(d) Any other event which effectively deprives the owner 75% of the value of all 1,208 pieces of logs, the shipment cannot
of the possession, at the port of destination, of the thing be said to have sustained a constructive total loss under Section
insured. (Section 130, Insurance Code). 139(a) of the Insurance Code.

A constructive total loss is one which gives to a person In the absence of either actual or constructive total loss,
insured a right to abandon, under Section 139 of the Insurance there can be no recovery by the insured Panama against the
Code. This provision reads: insurer, Oriental Assurance.

SECTION 139. A person insured by a contract of marine


insurance may abandon the thing insured, or any
particular portion thereof separately valued by the 43. G.R. no. L-54171 October 28, 1980
policy, or otherwise separately insured, and recover for
a total loss thereof, when the cause of the loss is a peril Jewel Villacorta v. Insurance Commission and Empire
injured against, Insurance Company

(a) If more than three-fourths thereof in value is Doctrine: Where the insured's car is wrongfully taken without
actually lost, or would have to be expended to recover it the insured's consent from the car service and repair shop to
from the peril; whom it had been entrusted for check-up and repairs
(assuming that such taking was for a joy ride, in the course of
(b) If it is injured to such an extent as to reduce its value which it was totally smashed in an accident), respondent
more than three-fourths; insurer is liable and must pay insured for the total loss of the
insured vehicle under the theft clause of the policy.
xxx xxx xxx
Facts: Ms. Villacorta took insurance in Empire Insurance In this case, evidence shows that the car was not merely taken
Company for her car, Colt Lancer, against loss or damage by for a “joy ride” or temporarily because the driver brought with
accidental collision, by fire, theft, and by malicious act. In the him grenade, 45 Colt.gun and other explosives, which to the
policy there includes an Authorized Driver Clause and Theft Court is an indication of a plan to permanently take the car.
Clause.
But Assuming, despite the totally inadequate evidence, that
On May 9, 1978, she brought her car to Sunday Machine the taking was "temporary" and for a "joy ride", the Court
Works for general check-up and repairs. On May 11, while in sustains as the better view that which holds that when a
the custody of the repair shop, the car was allegedly taken by person, either with the object of going to a certain place, or
6 persons and driven out to Rizal. While travelling, the car learning how to drive, or enjoying a free ride, takes possession
figured in an accident, hitting a gravel-sand truck parked at the of a vehicle belonging to another, without the consent of its
right side of the road. The car’s driver Benito Mabasa who is a owner, he is guilty of theft because by taking possession of the
personnel in the repair shop and one of the passengers died. personal property belonging to another and using it, his intent
And the car suffered extensive damage. For this, Ms. to gain is evident since he derives therefrom utility,
Villacorta claimed insurance for total loss of the car, but satisfaction, enjoyment and pleasure.
Empire Insurance Comp. denied the claim.

Ms. Villacorta brought the case before the Insurance


Commission (IC). The IC ruled in favor of insurance company
on the ground that neither the theft clause will apply because
the car was not stolen nor the authorized driver clause will
apply because the driver was not authorized by the insured.

Villacorta appealed the case.

Issue: Whether or not she is entitled to claim insurance on the 44. G.R no. L-36480, May 31, 1988
ground that the Theft Clause applies.
ANDREW PALERMO vs. PYRAMID INSURANCE CO., INC.
HELD: YES. When a car is unlawfully taken by some people,
be they employees of the car shop or not to whom it had been Facts: Mr. Palermo took a motor vehicle insurance from
entrusted, it is the theft clause, not the authorized driver clause Pyramid Insurance Co. In April, 1978, the car driven by Mr.
that applies. There is theft when a person unlawfully takes the Palermo with his father as passenger, caught an accident. As
property of another, without latter’s consent, with intent to gain. a result of which, the Nissan Sedan car was totally wrecked,
while Mr. Palermo sustained injuries and his father died. The
insurance company denied the claim for insurance because at 703 SCRA 314; Aug. 7, 2013
the time of the accident, the Mr. Palermo (insured) was driving Mendoza, J.
with an expired driver’s license, which means that the
Authorized Driver clause will not apply. DOCTRINE:

Issue: Interpretation of the Authorized Driver Clause –Does it FACTS: Fire Insurance policy worth 15 Million pesos insuring
require that the insured must have a valid license? machineries expressly stipulated to be located at Sanyo
Building was procured by PAP Co. Ltd, its owner, from Malayan
Held: No. Insurance, and named as beneficiary the Rizal Commercial
Banking Corp (RCBC), its mortgagee. The policy was renewed
There is no merit in the appellant's allegation that the plaintiff on “as is basis” up to May 13, 1998. During the subsistence of
was not authorized to drive the insured motor vehicle because the renewal, the insured machineries were transferred to Pace
his driver's license had expired. The driver of the insured Factory without consent of Malayan Insurance. Then, the same
motor vehicle at the time of the accident was, the insured got totally lost by fire. So PAP Co. claimed for insurance. But
himself, hence an "authorized driver" under the policy. Malayan Insurance Co. denied the claim on the ground that at
the time of loss, the insured machineries were transferred to a
While the Motor Vehicle Law prohibits a person from operating location different from that indicated in the policy. In other
a motor vehicle on the highway without a license or with an words, the transfer resulted to concealment, misrepresentation
expired license, an infraction of the Motor Vehicle Law on the and breach of an affirmative warranty under the renewal policy.
part of the insured, is not a bar to recovery under the PAP responded by saying that a notice was sent to Malayan by
insurance contract. It however renders him subject to the RCBC prior to the transfer made.
penal sanctions of the Motor Vehicle Law.
RTC Decision- Malayan is liable to pay because although there
The requirement that the driver be "permitted in accordance was a change in the condition of the thing insured as a result of
with the licensing or other laws or regulations to drive the the transfer of the machineries to another location, Malayan
Motor Vehicle and is not disqualified from driving such motor Insurance failed to show proof that such transfer resulted in the
increase of the risk insured against. In the absence of proof that
vehicle by order of a Court of Law or by reason of any
the alteration of the thing insured increased the risk, the contract
enactment or regulation in that behalf," applies only when the
of fire insurance is not affected per Article 169 of the Insurance
driver" is driving on the insured's order or with his permission."
Code.
It does not apply when the person driving is the insured
himself. CA Decision- Affirmed the RTC decision, but deleted the award
for attorney’s fees.
Malayan Insurance Corp v. PAP Co. Ltd. (Phil. Branch)
FIRE INSURANCE; CONCEALMENT
ISSUE: WON Malayan Insurance is liable to indemnify PAP Co. Under Section 27 of the Insurance Code, "a concealment
Ltd. for the fire insurance claim entitles the injured party to rescind a contract of insurance."

HELD: No. The policy forbade the removal of the insured Moreover, under Section 168 of the Insurance Code, the insurer
properties unless sanctioned by Malayan. Evidently, by the is entitled to rescind the insurance contract in case of an
clear and express condition in the renewal policy, the removal alteration in the use or condition of the thing insured. Section
of the insured property to any building or place required the 168 of the Insurance Code provides, as follows:
consent of Malayan. Any transfer effected by the insured,
without the insurer’s consent, would free the latter from any Section 68. An alteration in the use or condition of a thing
liability. insured from that to which it is limited by the policy made without
the consent of the insurer, by means within the control of the
The respondent failed to notify, and to obtain the consent of, insured, and increasing the risks, entitles an insurer to rescind
Malayan regarding the removal a contract of fire insurance.

The records are bereft of any convincing and concrete evidence Accordingly, an insurer can exercise its right to rescind an
that Malayan was notified of the transfer of the insured insurance contract when the following conditions are present, to
properties from the Sanyo factory to the Pace factory. What wit:
PAP did to prove that Malayan was notified was to show that it
relayed the fact of transfer to RCBC, the entity which made the 1) the policy limits the use or condition of the thing
referral and the named beneficiary in the policy. Malayan and insured;
RCBC might have been sister companies, but such fact did not
make one an agent of the other. The fact that RCBC referred 2) there is an alteration in said use or condition;
PAP to Malayan did not clothe it with authority to represent and
bind the said insurance company. After the referral, PAP dealt 3) the alteration is without the consent of the insurer;
directly with Malayan.
4) the alteration is made by means within the insured’s
Malayan is entitled to rescind the insurance contract control; and

the transfer of the location of the subject properties, without 5) the alteration increases the risk of loss.20
notice and without Malayan’s consent, after the renewal of the
policy, PAP clearly committed concealment, misrepresentation In the case at bench, all these circumstances are present. It was
and a breach of a material warranty. Section 26 of the Insurance clearly established that the renewal policy stipulated that the
Code provides: insured properties were located at the Sanyo factory; that PAP
removed the properties without the consent of Malayan; and
Section 26. A neglect to communicate that which a party knows that the alteration of the location increased the risk of loss.
and ought to communicate, is called a concealment.
against it, because they are not parties to the insurance
contract.
ISSUE: WON a victim resulting from tort / quasi-delict can
directly sue the insurer
HELD: YES. It is settled that where the insurance contract
provides for indemnity against liability to a third party, such third
party can directly sue the insurer. The liability of the insurer to
such third person is based on contract while the liability of the
insured to the third party is based on tort. First Insurance cannot
evade its liability as insurer by hiding under the cloak of the
insured. Its liability is primary and not dependent on the
recovery of judgment from the insured.
The injured for whom the contract of insurance is intended can
First Integrated Bonding & Insurance Co. Inc. v. sue directly the insurer. The general purpose of statutes
Hernandez (COMPULSARY THIRD PARTY LIABILITY) enabling an injured person to proceed directly against the
199 SCRA 746, July 31, 1991 insurer is to protect injured persons against the insolvency of
Medialdea, J. the insured who causes such injury, and to give such injured
person a certain beneficial interest in the proceeds of the policy,
DOCTRINE: and statutes are to be liberally construed so that their intended
FACTS: Silverio Blanco, owner of a jeepney, procured a purpose may be accomplished. It has even been held that such
compulsory third party liability insurance from First Insurance. a provision creates a contractual relation which inures to the
His jeepney driven by him got involved in an accident which benefit of any and every person who may be negligently injured
caused the death of a 3-yr old girl. The parents of the deceased by the named insured as if such injured person were specifically
victim filed a complaint for damages against Blanco and First named in the policy.
Insurance. Judgment was rendered in favor of the victim,
ordering Blanco and First Insurance to solidarily pay the award. Compulsory Motor Vehicle Liability Insurance (third party
Then judgment was entered. First Insurance filed a petition for liability, or TPL) is primarily intended to provide compensation
relief from judgment, but denied. First Insurance also filed a for the death or bodily injuries suffered by innocent third parties
Motion for Recon of the decision, but was also denied. or passengers as a result of a negligent operation and use of
Aggrieved, First Insurance filed a Petition for Certiorari with the motor vehicles. The victims and/or their dependents are
SC contending that the victim’s family has no cause of action assured of immediate financial assistance, regardless of the
financial capacity of the motor vehicle owners.
. . . the insurer's liability accrues immediately upon the
occurrence of the injury or event upon which the liability
depends, and does not depend on the recovery of judgment by
the injured party against the insured

Das könnte Ihnen auch gefallen